Law Test 1

Pataasin ang iyong marka sa homework at exams ngayon gamit ang Quizwiz!

White-collar crimes are often committed by employees against a business.

Answer: TRUE

Which of the following is the application of ethics to special problems and opportunities experienced by those in business? A) Situational ethics B) Consequentialism C) Business ethics D) Sarbanes-Oxley principles E) Business utilitarianism

C

For an intentional tort to occur, the plaintiff must show the common element of ________. A) negligence B) mistake C) intent D) carelessness E) There is no common element for all intentional torts.

C

Which of the following is not one of the five steps to dispute resolution outlined by the United Parcel Service? A) Open door policy B) Facilitation C) Management review D) Peer review E) Mandatory mediation

C

Which of the following is reported in the Federal Reporter? A) Statutes B) Executive orders C) Common law D) Administrative laws E) Local ordinances

C

For diversity-of-citizenship, which of the following is needed? A) Only that the plaintiff not reside in the same state as the defendant. B) Only that the plaintiff reside in the same state as the defendant. C) Only that the controversy concern an amount in excess of $75,000. D) Only that the controversy concern an amount in excess of $100,000. E) That the plaintiff and the defendant reside in different states and that the controversy concerns an amount in excess of $75,000.

e

The state of Tennessee is getting ready to pass legislation that impacts businesses within its state and is not addressed by powers in the Constitution. Which amendment is implicated? A) First B) Second C) Fifth D) Sixth E) Tenth

e

Which type of tort is the most willful of torts? A) Intentional torts. B) Strict liability torts. C) Negligence torts. D) Assault and battery. E) Slander.

A

Critics of the adversarial system cite time and expense of a lawsuit along with damage to reputations as some of the drawbacks of litigation.

Answer: TRUE

Defamation can be a civil or criminal action in China.

Answer: TRUE

Embezzlement is distinguished from larceny because the embezzler does not take the property from another and is already in possession of the property.

Answer: TRUE

51) Which of the following is true regarding awards of punitive damages in the U.S.? A) It is not always easy to predict what a court will do regarding punitive damages in any given case. B) Punitive damages may not be awarded in excess of $3 million. C) A ratio of punitive damages to compensatory damages greater than 9 to 1 violates due process. D) The Supreme Court ruled that the issue of punitive damages is for the jury and will not be disturbed on appeal, regardless of the ratio of punitive damages to compensatory damages. E) Awards of punitive damages in the U.S. are very similar to awards of punitive damages under the Canadian judicial system.

A

A ________ allows the parties to submit a specific dispute involving a contractual provision to arbitration if a contract does not provide for arbitration. A) submission agreement B) binding mediation clause C) binding arbitration clause D) resolution submission clause E) Courts do not allow parties to submit anything if the contract does not provide for arbitration

A

A ________ trial is an abbreviated trial that leads to a non-binding jury verdict. A) summary jury B) jury stipulation C) verdict of the jury D) completed jury E) mediation jury

A

A business is upset about the tactics that a competitor is employing to gain business and sues the business. The plaintiff was not necessarily harmed in any way, but the court finds that a tort was committed and awards the plaintiff $10 in damages. This $10 in damages is an example of what type of damages? A) Nominal B) Compensatory C) Punitive D) Retaliatory E) Revenge

A

A company has to dispose of bio-hazardous waste products. Dropping them in the landfill anonymously would be the cheapest solution, but the company does not do this because it is frightened of the bad publicity which could result if it was found out. In considering the possible complicity, the company's decision was shaped by the: A) Public disclosure test B) Profit motive C) Golden rule D) WPH Method E) Universalization

A

A defendant can use the ________ defense if the idea for a crime originated not with the defendant but with a police officer or other government official who suggested it to the defendant, and the defendant would not otherwise have committed the crime. A) Entrapment B) Necessity C) Justifiable use of force D) Duress E) Inescapable action

A

A manager may think it is wrong to terminate a person whose spouse has a terminal illness because a company has an absolute obligation to support its employees when they are vulnerable. With which ethical theory is the business manager acting consistently? A) Deontology B) Ethical relativism C) Act utilitarianism D) Ethical fundamentalism E) Rule utilitarianism

A

A reporter does not like a candidate for president and he publishes a made up story about the candidate having an affair. Did the reporter commit a tort? A) Yes, because the reporter acted with actual malice. B) No, because the candidate for president is a public figure. C) No, because the story was made up and unlikely to be believed. D) No, because having an affair would not harm the reputation of a presidential candidate. E) No, because politicians cannot be defamed.

A

A very wealthy defendant intentionally assaults his butler. The butler sues and the wealthy defendant is found to have committed a tort. The court decides to award punitive damages. In determining the amount of punitive damages, what are some factors that will be taken into account? A) The severity of the wrongful conduct and the wealth of the defendant. B) The fact the defendant is wealthy and that the victim was an employee. C) The severity of the defendant's conduct and the fact the victim was an employee. D) The length of service the butler provided to the defendant and the wealth of the defendant. E) The wealth disparity between the butler and the wealthy defendant.

A

Med-arb is which type of dispute resolution process? A) A process in which the parties agree to start out in mediation and, if the mediation is unsuccessful on one or more points, to move on to arbitration. B) A process in which the parties agree to start out in arbitration and, if the arbitration is unsuccessful on one or more points, move on to court-annexed ADR. C) A process in which the parties agree to start in mediation and move to litigation if the mediation is unsuccessful. D) A process in which the parties agree to start in arbitration and move to litigation if the mediation is unsuccessful. E) A special type of blended mediation and arbitration applicable only in the medical malpractice arena.

A

The "public disclosure" test for ethical behavior is sometimes referred to as the ________ test. A) Television B) Powell C) Self-conscious D) Golden E) Primary

A

Which of the following is true regarding the use of legal counsel at arbitration proceedings? A) It is up to each of the parties. They may have legal counsel or they may not. B) Parties may not have legal counsel. C) Parties may have legal counsel if they had legal counsel in mediation proceedings. Otherwise, legal counsel may not be used. D) Individuals may have legal counsel, but corporations may not. E) Corporations may have legal counsel, but individuals may not.

A

Corporations cannot be held criminally accountable for strict-liability offenses under current law.

Answer: FALSE

"Legal positivism" is also known as "natural law."

Answer: FALSE

A private nuisance occurs when a person uses their property in a manner that harms the public as a whole.

Answer: FALSE

A strict-liability tort occurs when a defendant acts with the intention of engaging in a specific act that ultimately results in injury.

Answer: FALSE

A summary jury trial leads to a binding jury verdict.

Answer: FALSE

) Jill and Joe are in business and designed a new technology in which they received a patent from the U.S. Patent and Trademark Office. Both Jill and Joe are New Mexico residents but spend the summer months in Minnesota. They find out a competitor has violated their patent. In which court would they want to bring a lawsuit? A) The New Mexico state court only B) The Federal District Court C) The Minnesota state court only D) Either the New Mexico or Minnesota state court E) Either a state or a federal court

B

The misappropriation of trust funds or money held in a fiduciary capacity is which of the following? A) False entries B) Defalcation C) False pretenses D) Ponzi scheme E) Pretexting

B

A benefit of arbitration is which of the following, according to the text? A) Arbitrators are assigned so parties do not have to pick them. B) Arbitration is generally less expensive than litigation. C) Arbitrators are bound by the same rules as judges in applying precedent. D) Arbitrations can be set aside on appeal for the same reasons as court judgments. E) Arbitrations prohibit the use of legal counsel.

B

Which of the following is not true in regards to the innocent dissemination defense of online service providers (OSPs) in Canada? A) There are no landmark cases or any legislation clearly establishing liability of OSPs. B) The defendant paid for the information that they authored or printed. C) The defendant does not know of the libel contained in the work published or authored by them. D) There was no reason for the defendant to suppose the work he or she authored or published would be libelous. E) It was not negligence on the defendant's part that he or she did not know that the work contained libelous material.

B

Which of the following is punishable by imprisonment for more than one year or death? A) Misdemeanor. B) Felony. C) Petty offenses. D) Tort offenses. E) Any business related crime.

B

Which of the following is true of a negligence tort? A) A defendant commits a negligence tort when he subjects someone to a reasonable risk of harm. B) A defendant commits a negligence tort when he is careless to someone else's detriment. C) A defendant commits a negligence tort when he forms intent to cause harm but causes a different type of harm. D) A defendant commits a negligence tort when he accidentally engages in a criminal act and someone is hurt. E) A defendant commits a negligence tort only if he commits an inherently dangerous action which can never be safe.

B

A crime can be a felony, misdemeanor, or petty offense. How is this determination usually made? A) The judge decides. B) The jury decides. C) The statute defining the crime usually establishes whether the crime is a felony, misdemeanor, or petty offense. D) The prosecutor decides how the charges will read. E) The statute defining the crime usually establishes whether the judge or jury decides if the crime is a felony, misdemeanor, or petty offense.

C

A defendant is found liable for causing cancer due to the dumping of toxins into the river. The court awards a plaintiff who was sick $5 million in damages, in addition to compensating the plaintiff for all medical costs, lost wages, pain and suffering, and actual losses. The $5 million award is intended to ensure the company never dumps toxins again. The $5 million is an example of what type of damages? A) Nominal B) Compensatory C) Punitive D) Retaliatory E) Revenge

C

A defendant uses a[n] ________ when her or his answer admits that the facts contained in the complaint are accurate but also includes additional facts that justify the defendant's actions and provide a legally sound reason to deny relief to the plaintiff. A) Secondary answer B) Pleading defense C) Affirmative defense D) Formal answer E) Personam answer

C

Which of the following is true about search warrants under the Fourth Amendment? A) If a police officer believes it is likely that items sought will be removed from the premises, the officer must obtain an Urgent Search Warrant prior to the search. B) A government official must always obtain a search warrant prior to engaging in a search of corporate property. C) A warrantless search is permissible when an industry has a long history of pervasive regulation, such as a search authorized by the Federal Mine Safety and Health Act. D) A police officer must always obtain a search warrant before searching a vehicle for stolen goods. E) The rule on "unreasonable search and seizure" only applies to individual citizens, not corporations.

C

Which of the following is true regarding corporate executives being held personally liable for business crime under the "responsible corporate officer" doctrine? A) A corporate executive may never be held personally liable for a business crime. B) Corporate executives may only be held personally liable for a business crime if they benefited personally from their actions in executing the business crime. C) Corporate executives may be found personally liable for a business crime even if they did not engage in, direct, or know about a specific criminal violation. D) A court may not assess criminal liability on a corporate executive unless the executive directly engaged in the specific criminal violation. E) Criminal liability may not be assessed against a corporate executive unless the executive directly engaged in the criminal activity and also profited directly and personally from the criminal activity.

C

) If a defendant fails to respond to the plaintiff's complaint with an answer when the plaintiff's complaint alleges facts to support a response, the plaintiff could ask the court for a(n) ________. A) answer on the pleadings B) judgment for cause C) final judgment D) default judgment E) entry judgment

D

A company pays bribes in foreign countries in order to conduct operations there. What legal consequences can the company face? A) No legal consequences as long as the bribe was legal in the country where it was paid. B) No legal consequences as long as the company was acting in the best interests of shareholders. C) No legal consequences as long as the company had no choice but to pay the bribe if it wished to do business in the foreign country. D) Legal consequences under the Foreign Corrupt Practices Act. E) Legal consequences under Glass-Steagall

D

A false statement of a material fact regarding ownership of business property that results in a loss of sales for the business is referred to as ________. A) Slander of quality B) Trade libel C) Libel of title D) Slander of title E) Libel of sale

D

A full bench for the United States Supreme Court consists of how many justices? A) 10 B) 8 C) 12 D) 9 E) 7

D

There are ________ branches of government laid out in the U.S. Constitution. A) six B) five C) four D) three E) two

D

Which of the following is true regarding activities of the Public Company Accounting Oversight Board? A) The Board ensures that auditors and public accounting firms compile accurate and truthful financial reports for the companies they audit, but it has no responsibility in regard to whistle-blowing protection. B) The Board requires that companies devise a system that allows employees to report suspicions of unethical behavior, but it does not address financial reports prepared by auditors and public accounting firms. C) The Board requires that the universalization test be used as the primary ethical guideline and also mandates protection for whistle-blowers who report suspicious activity involving financial reports. D) The Board ensures that auditors and public accounting firms compile accurate and truthful financial reports for the companies they audit and also requires that companies devise a system that allows employees to report suspicions of unethical behavior. E) The Public Company Accounting Oversight Board was abolished during the Obama administration and no longer exists.

D

Which of the following is true regarding the form of ADR known as "negotiation"? A) The ADR process known as "negotiation" is the same as the ADR process known as "mediation." B) There is only one approach to negotiation, problem-solving. C) There is only one approach to negotiation, adversarial. D) There are two approaches, adversarial negotiation and problem-solving negotiation. E) There is no form of ADR known as negotiation.

D

Which of the following is true regarding the groups generally considered to be public figures for purposes of the public figure privilege to actions for defamation? A) Politicians, but not entertainers, are generally considered to be public figures. B) Entertainers, but not politicians, are generally considered to be public figures. C) Small business owners and politicians, but not entertainers, are generally considered to be public figures. D) Politicians and entertainers, but not small business owners, are generally considered to be public figures. E) Small business owners, but not politicians or entertainers, are generally considered to be public figures.

D

Which of the following properly notifies a defendant that they are being sued and explains how and when to respond to a complaint? A) Answer B) Service document C) Court report D) Summons E) Court docket

D

After voir dire, which is the next logical progression in a typical lawsuit? A) Examination of witnesses. B) Pretrial conference. C) Cross-examination of witnesses. D) Reading of the jury instructions. E) Opening statements.

E

) If a state passed a law prohibiting the use of contraceptives, would a court likely find this law to be unconstitutional? A) Yes, unless the law can pass the rational-basis test. B) No, as long as the law treats citizens from other states the same as citizens in that state. C) No, as long as the law does not discriminate against women. D) Yes, because it is an unreasonable seizure of property. E) Yes, because it violates individuals' right to privacy.

E

Which statement about the ability of individual states to collect sales tax on Internet transactions is true? A) Individual states may, without restriction, collect sales tax from merchants on items purchased through the Internet by in-state residents. B) Only with permission of Congress, granted on a state-by-state basis, may individual states collect from merchants sales tax on items purchased through the Internet by in-state residents. C) States cannot collect sales tax on Internet sales from merchants nor may a state require that its citizens report Internet purchases so that they may be taxed. D) A state cannot collect sales tax from merchants unless sales to the state's citizens amount to more than 1 million in the aggregate. E) A state can collect sales tax from a merchant only if the merchant has a store or distribution center in the state.

E

Which statement is true regarding Liz's actions toward Min-ji? A) Liz committed an assault. B) Liz committed a battery. C) Liz committed both an assault and a battery. D) Liz did not commit any offense because she did not actually hurt Min-ji. E) Liz did not commit any offense because she was acting in the defense of Daniel.

E

Why must federal legislation that affects business be based on an expressed constitutional grant of authority? A) Because the U.S. Congress passed a law to that effect. B) Because the federal government has only those powers granted to it by the Constitution. C) Because President Lincoln issued an executive order to that effect. D) Because President Washington issued an executive order to that effect. E) Because it is a custom.

b

) According to the principle of federalism established by the U.S. Constitution, how is the authority to govern allocated? A) It is allocated to the federal government. B) It is allocated to the states. C) It is allocated to local jurisdictions. D) It is allocated to the U.S. Senate. E) It is divided between federal and state governments.

e

DeRay and Jenna live in different states. DeRay is suing Jenna for injuring him by causing him to fall. He is seeking $20,000 in damages. DeRay can sue in: A) Federal court only, because diversity jurisdiction exists. B) Federal court only, because DeRay and Jenna live in different states. C) Federal court only because the amount in controversy exceeds $10,000. D) Both federal and state court. E) The state where the fall took place.

e

Congress can enact laws, but the president can then veto any law that Congress passes.

Answer: TRUE

Congress can use its spending power to achieve social welfare objectives.

Answer: TRUE

Contracts, employment decisions and payments to suppliers are protected by business law.

Answer: TRUE

(The) ________ maintains that the right course of action is the option most consistent with the building and maintaining of human relationships. A) Ethics of care B) Principle of virtue ethics C) Deontology D) Act utilitarianism E) Rule utilitarianism

A

) If someone has failed to live up to a promise made to a business organization, the business organization should: A) Seek a legal remedy within the courts B) Ask the lawmakers to impose a punishment upon the violation C) Publicly shame the violator for breaking his promise D) Turn to the executive branch to impose legal penalties E) Ask lawmakers and the executive branch to impose consequences

A

) In the case of Robert Gilmer v. Interstate/Johnson Lane Corporation involving whether a claim under the federal Age Discrimination in Employment Act was subject to arbitration, what did the U.S. Supreme Court rule? A) That the agreement by which the plaintiff agreed to arbitrate all employment disputes as a condition of his employment was valid. B) That the text of the federal Age Discrimination in Employment Act prohibited arbitration. C) That the EEOC would be bound by the employee's agreement to arbitrate all employment disputes and that, therefore, requiring arbitration would be improper. D) That compulsory arbitration improperly deprived the plaintiff of a public forum through which to prove his claims. E) That arbitration was an inadequate remedy because arbitrators may not fashion equitable remedies.

A

) Louisiana passes a law that changes the voting age and requirements for its citizens to vote. Which test will the courts use to evaluate the case? A) Strict scrutiny B) Intermediate scrutiny C) Rational basis scrutiny D) Popular scrutiny E) Legal scrutiny

A

) ________ is the type of law involved in comparing laws between the U.S. and Winland. A) Comparative law B) Contributory law C) Comprehensive law D) Complete law E) Delineated law

A

22) Which of the following do appellate courts primarily handle? A) Questions of law B) Questions of fact C) Questions of law and fact D) Cases when they initially enter the legal system E) Questions of law and fact, and also cases when they initially enter the legal system

A

A new type of hydraulic fracturing ("fracking") has been implemented to help bring more natural gas to a well site. The company associated with this process has followed all federal and state guidelines associated with the process of fracking. Residents however are very unhappy and vocal about the process of fracking and fights have broken out amongst the resident's and workers. Resident's demand the company be shut down. A new law was implemented that is designed to help keep the peace yet look at alternatives to the fracking process. The purposes of the new law include: A) Providing order, serving as a moral guide, serving as an alternative to fighting, facilitating a sense that change is possible B) Creating enforceable rules of law which change the way that the manufacturing process must be changed C) Providing new rules that would allow for civil disobedience D) Setting new rules and guidelines that would allow for protests to happen on the fracking site E) Guaranteeing the personal freedoms of the company to continue fracking as long as they were doing so within state and federal guidelines

A

A(n) ________ is an order of the court to the sheriff to sell personal assets of the defendant and turn over the proceeds, minus the expenses and fees, to a judgment holder. A) writ of execution B) writ of certiorari C) announcement of judgment D) penalty decree E) motion to collect

A

Aiden, who just started at his new job as an assistant manager at a fast-food restaurant, wants to make a good impression. He thinks that things are going well. On only the second day on the job, however, he sees his manager, Debbie, slink out of the restaurant with a big box of hamburgers. She puts them in her car and speeds away. Aiden is fairly certain that Debbie did not pay for the hamburgers. Aiden asks advice of his best friend, Kyle; his girlfriend, Mia; his sister, Katie; and his dad, Joe. Kyle says that there are no real objective standards and that Aiden should just decide to do what is best for himself. Mia says that Aiden should think about the consequences and focus on the greatest benefit to all involved. She believes that it will do no real harm for Aiden to keep his mouth shut because the fast-food restaurant is making plenty of money and probably does not pay Debbie enough anyway. Plus, getting in trouble with Debbie could cause problems at work, and if Aiden loses his job, Aiden and Mia might have to put off their impending marriage. Katie says that on balance, following the rule of producing the greatest good would mean that he should report Debbie to her boss because stealing does not generally produce the greatest satisfaction. Aiden's dad says that as a matter of principle, Aiden should report Debbie to her boss because stealing is simply wrong. The advice given by Joe most accurately fits which theory? A) Deontology B) Act utilitarianism C) Rule utilitarianism D) Ethical relativism E) Virtue ethics

A

An example of pursuing change through identification with the vulnerable would involve: A) Arguing for stronger laws to protect abused children because abused children cannot protect themselves B) Arguing for a stronger law to protect abused children because treating child victims is too costly for the public C) Arguing for a stronger law to protect abused children because those who abuse children are not obeying duly authorized laws on child protection D) Arguing for a stronger law to protect abused children because other jurisdictions have stronger laws E) Arguing that current laws on child abuse should not be changed because of the tradition behind the laws

A

Assume a judge writes that she is deciding to enforce a law in question but that her decision does not mean that she sees the law as the morally correct rule. The judge would have leanings in the direction of ________. A) Legal positivism B) Natural law C) Legal realism D) Conscience reaction E) Concluding law

A

At trial a defense lawyer states that evidence should not be admitted because it is "fruit of the poisonous tree." Upon which of the following is the attorney relying? A) The exclusionary rule. B) The Fourth Amendment Rule. C) The Eden Rule. D) The Adam Rule. E) The Eve Rule.

A

Brandon installs two-way mirrors in the dressing room of his ladies' clothing store. He secretly uses the mirrors to watch women while they try on clothing. Which of the following torts has Brandon committed? A) Intrusion on an individual's affairs or seclusion. B) Assault. C) Public disclosure of private facts. D) False light. E) Commercial appropriation.

A

By providing information to Claire regarding the purchase, Eva is guilty of which offense, if any? A) She is guilty of an insider-trading violation by tipping the information. B) She is guilty of an insider-trading violation by being tipped. C) She is guilty of an insider-trading violation by both tipping and being tipped. D) She is guilty of criminal extortion. E) She is not guilty of any offense.

A

By purchasing stock in her company, Eva is guilty of which offense? A) Insider trading B) Tipping C) Insider misappropriation D) Extortion E) She is guilty of no offense

A

Carlos, who has a large trust fund, is angry with Isabel because she stopped dating him. Isabel ran a successful dog grooming shop, but Carlos decided to open a dog grooming shop next door and charge less than it would take to make a profit, solely to run her out of business. Which of the following is correct? A) Carlos has committed the tort of unfair competition. B) Carlos has committed the tort of interference with contract. C) Carlos has committed the tort of unfair competition and also the tort of interference with contract. D) Carlos has committed the tort of disparagement. E) Carlos has not committed any tort.

A

Cooper sues Company A in state court in South Carolina, where he lives, for negligence alleging personal injury and property damage of $100,000 after a truck driven by an employee of Company A rear-ended his pickup truck. Company A is incorporated in Delaware, has its headquarters in New York, but does a substantial amount of business in South Carolina. Claiming diversity of citizenship, Company A seeks removal to federal district court, but Cooper opposes the motion. Which of the following is true regarding whether the case may be properly removed to federal district court? A) The amount in controversy satisfies diversity requirements; and if Company A's nerve center is in a state other than South Carolina, then the case may be properly removed to federal court. B) The amount in controversy satisfies diversity requirements; and because Company A is incorporated and has its headquarters in a state other than South Carolina, the case may be properly removed to federal court. C) Because the amount in controversy satisfies diversity requirements and Company A is incorporated in a state other than South Carolina, the case may be properly removed to federal court regardless of where Company A's headquarters, nerve center, or principal place of business is located. D) Because the amount in controversy satisfies diversity requirements and Company A is headquartered in a state other than South Carolina, the case may be properly removed to federal court regardless of where Company A is incorporated and regardless of the location of its nerve center. E) Because the amount in controversy fails to satisfy jurisdictional requirements, regardless of the location of Company A, the case may not be removed to federal court.

A

Creditors sued Hulio and Juanita and obtained a judgment in California. The couple, however, left the state and have resided in Nevada for the past two years. Would a court in Nevada uphold the California judgment? A) Yes, under the full faith and credit clause. B) Yes, under the commerce clause. C) Yes, under the preemption doctrine. D) No, because each state has the right not to honor a judgment from another state. E) No, because this would be a federal case not a state case.

A

DeShaun, who is a bit eccentric, is fed up with the way a certain employer in his town treats employees and decides to sue on behalf of all those employees. DeShaun also says that he is going to start his case at the appellate court level, skipping over all those "lesser" judges. DeShaun says that those justices will surely hear him out and that he will also seek a jury. Assuming DeShaun eventually ends up at the appellate level, is he entitled to a jury at the appellate level? A) No, jurors are not used at the appellate court level. B) Yes, but only if he had a jury first at the trial court level. C) Yes, if he satisfies the requirements to initially bring the case in appellate court, he is entitled to a jury. D) Yes, but only if the employer agrees to a jury. E) Yes, because appellate courts decide questions of fact while questions of law are decided at the trial court level

A

Denise McCrae is a school-board member in Mariposa County. Appalled by the amount of junk food in the schools, she convinces the state legislature to pass an ordinance that no stores may sell any unhealthy junk food within 600 feet of any school or daycare center. A political science teacher questions whether the regulation is valid and is told that it is a valid exercise of the jurisdiction's police power to protect its citizens. Convenience stores balk at the regulation and challenge it in court claiming that the law impermissibly affects interstate commerce. 24) Which of the following is the best definition of the term "police power"? A) The residual powers retained by each state to safeguard the health and welfare of its citizenry. B) The residual powers retained by the federal government to enforce valid laws and regulations. C) The powers granted to local government to enter residences without a search warrant. D) The powers granted to state governments to tax for the purposes of having law enforcement. E) The powers granted to state government to imprison citizens of other states who commit crimes within a state.

A

Dr. Tanaka was served with a malpractice lawsuit based on allegedly removing the wrong organ. Following discovery, it becomes clear that the plaintiff has no credible evidence against the doctor, and that no reasonable jury could rule in the plaintiff's favor. How should Dr. Tanaka's lawyer proceed? A) The lawyer should make a motion for summary judgment. B) The lawyer should make a motion for additional discovery. C) The lawyer should make a motion to amend the answer in order to make a motion to dismiss. D) The lawyer should file a motion for judgment on the pleadings. E) The lawyer should proceed to trial.

A

Drake and Julie are discussing criminal law. Julie tells Drake that, ordinarily, law enforcement agents must demonstrate that there is probable cause that a suspect committed or is planning to commit a crime in order to obtain an arrest warrant. Is she correct? A) Yes B) Yes, but only if the suspect agrees C) No, law enforcement never needs a warrant D) No, law enforcement only needs to have reasonable suspicion E) No, law enforcement can make a judgment call at any time to arrest a person

A

Economist Milton Friedman argued that the only social responsibility of business is: A) to increase profits B) to do right by others C) to follow the law D) to increase shareholder values E) to not harm the environment

A

For a court to be able to provide a ruling in the case, there must be an adverse relationship between the parties, the action must give rise to an actual legal dispute and the courts must have the ability to render the decision that will resolve the dispute and not be a hypothetical situation. This is known as ________. A) case or controversy B) mootness C) ripeness D) venue E) compulsory

A

Georgena believes that her company cannot force her to say prayer each morning before the work day starts. Which amendment to the Constitution would she look to for guidance? A) First B) Second C) Fifth D) Ninth E) None, Georgena would have to do as her company requested or risk being fired.

A

If Maria decides to follow all laws and regulations, even if she does not agree with them, this approach is rooted in what? A) Legal positivism B) The historical school C) Legal realism D) Natural law E) Cost-benefit analysis

A

If a business decides to act outside of the prevailing ethical norms of that community: A) It risks damage to its reputation and ill will. B) There really is no risk to businesses since the community really needs the business to survive. C) It only has to worry about the legal implications of its actions. D) It takes the chance the law will impose fines against it. E) There should be no concerns because the business can move to another community if it needs to.

A

If a plaintiff files a case in state court that could also have been filed in federal court, does the defendant have any say in where the case is heard? A) The defendant has a right to move the case to federal court. B) The defendant can have the case moved to federal court only if federal jurisdiction question is involved. C) The defendant can have the case moved to federal court only if the state trial court judge grants permission at his or her discretion. D) The defendant can have the case moved to federal court only if the plaintiff's filing expenses in state court are paid by the defendant. E) The defendant has no choice, and the case will stay in state court.

A

If a victim of assault does not want the abuser arrested but the prosecutor brings charges anyway since the assault violated criminal laws, the reason for the prosecutor's actions is: A) A violation of criminal laws is an act against the public as a whole. B) The abuser has committed a civil law violation in addition to a criminal one. C) Private law has been violated and the prosecutor has a duty to act. D) Natural law has been violated and the prosecutor has a duty to act. E) The prosecutor's responsibility is to regulate disputes between private individuals under criminal law

A

If after reviewing the pleadings, the judge determines that the only reasonable decision is in favor of the moving party, the court may grant which of the following? A) Motion for judgment on the pleadings B) Motion for summary judgment C) Motion for sanctions D) Motion for discovery E) Motion for production

A

In South Dakota v. Dole, the validity of a federal statute granting federal funds for state highways to only those states in which 21 is the legal drinking age was questioned. What did the U.S. Supreme Court rule? A) The Court upheld the statute. B) The Court upheld the statute only in the case of states that could not provide matching funds; and if matching funds could be provided, the Court ruled that there was insufficient reason for the law. C) The Court upheld the statute only in the case of states that could provide matching funds; and if matching funds could not be provided, the Court ruled that Congress was overreaching its power in regard to partnership between state and federal government. D) The Court struck the statute on the basis that it violated the commerce clause under the U.S. Constitution. E) The Court struck the statute on the basis that it violated the taxing and spending powers of the federal government under the U.S. Constitution.

A

In an early U.S. Supreme Court interpretation of the commerce clause, the Court ruled that ________ could regulate labor relations at a manufacturing plant because a work stoppage at the plant would seriously impact interstate commerce. A) Congress B) the President C) the Senate D) state legislatures E) state governors

A

In order for a federal court to hear a case involving a federal question, the amount in controversy must be at least ________. A) No amount is needed B) $75,000 C) $80,000 D) $90,000 E) $100,000

A

In the Case Opener, a defendant sought to dismiss a plaintiff's case against Accutane arising from allegations that Accutane caused ulcerative colitis and irritable bowel syndrome. Which of the following court rulings was the result of the defendant's motion to dismiss based on statute of limitation grounds? A) That because the plaintiff was not warned by her doctor of specific risks caused by the medication, the statute of limitations did not begin to run until she became aware of an advertisement discussing the risk. B) That because the plaintiff was warned by her doctor of specific risks caused by the medication, the statute of limitations began to run prior to the time she became aware of an advertisement discussing the risk; and the statute of limitations barred her claim. C) That the statute of limitations began to run on the plaintiff's claim when the risk of the drug was discovered by the manufacturer, not on the date on which the plaintiff became aware of the risk and that the statute of limitations, therefore, barred her claim. D) That in lawsuits involving this type of drug, there is no statute of limitations because each day the plaintiff suffers from pain results in the beginning of a new cause of action. E) That the defendant committed fraud preventing it from relying on the statute of limitations.

A

In which school of jurisprudence is the concept of stare decisis rooted? A) The historical school B) Legal realism C) The cost-benefit analysis school D) Positivism E) Cultural reenactment

A

Jill hires Bob to deliver flowers to local customers. If Bob and Jill get into a dispute in regards to Bob's expenses and salary, what type of law would govern the dispute? A) Private B) Joint C) Criminal D) Public E) Common law

A

Kantian ethics is associated with which of the following ethical theories? A) Deontology B) Act utilitarianism C) Rule utilitarianism D) Absolutism E) Situational ethics

A

Which tort occurs when one person places another in fear or apprehension of an immediate, offensive bodily contact? A) Assault B) Battery C) Assault and battery D) Defamation E) Strict liability

A

Kimora, a licensed veterinarian, was recently elected to the U.S. Senate. Kimora is very concerned about the lack of care for homeless cats and dogs. She proposes legislation whereby citizens of all states are to be taxed sufficiently to provide for no-kill animal shelters, and that citizens in states with higher than average incomes are to be taxed at a higher rate than citizens of other states. She also proposes that federal funds for state highways be denied to any state that fails to provide assistance to low-income citizens in covering pet vaccinations. José, another new senator, tells Kimora that, apart from the mandated federal income tax, Congress lacks the authority to tax states because the U.S. Constitution expressly reserves that right to the states. Sven, another senator, tells Kimora that Congress has no authority to link highway funds or any other funds with social welfare objectives. Devonne, a clerk in the Senate, tells Kimora that Congress is absolutely prohibited from taxing residents of one state at a higher rate than citizens of another state. 31) In saying that Congress lacks the authority to tax states, is José correct? A) He is incorrect. The U.S. Constitution contains no such reservation. B) He is incorrect because while the U.S. Constitution prohibits taxation in areas in which states also tax, most states are not taxing for pet care purposes at this time. C) He is incorrect in saying that the U.S. Constitution generally reserves the right to tax the states but he is correct in saying that Congress lacks the authority to tax in this manner because Congress cannot lay taxes for any reason other than essential needs of the federal government. D) He is incorrect in saying that the U.S. Constitution generally reserves the right to tax the states but he is correct in saying that Congress lacks the authority to tax in this manner because the president has issued an executive order prohibiting such taxation. E) He is incorrect because the prohibition against Congress imposing taxes apart from the federal income tax is contained in a federal statute passed by Congress, not in the U.S. Constitution.

A

Lizzy and her boyfriend Bobby are at the local restaurant. The two have been arguing and Lizzy says she has had enough and turns to leave. Bobby, who is substantially bigger than Lizzy, draws back his hand as if to hit Lizzy. Lizzy turns around and ducks just in time to avoid Bobby's punch. What, if any, tort can Lizzy sue for? A) Lizzy could sue for assault. B) Lizzy could sue for battery. C) Lizzy could sue for assault and battery. D) Lizzy could sue for negligence since Bobby missed punching her. E) Lizzy could not sue for a tort as no harm came from his threatening to punch her.

A

Mark and Elizabeth have a dispute over who breached a contract between their respective companies. The two hire a referee to offer a legally binding judgment in a dispute. What form of ADR is this? A) Private trial B) Summary jury trail C) Negotiated settlement D) Mediation E) Arbitration

A

Min-ji's action in jabbing Daniel after she missed his nose constitutes which of the following? A) Battery. B) Nothing, because his conduct would not be deemed offensive. C) Nothing, because she missed the nose, and Daniel was not actually hurt. D) Nothing, because by entering Min-ji's yard, Daniel consented to any offensive touching. E) Nothing, because Daniel deserved what he got.

A

Penny and Theodore comprise a scheme whereby Penny will send information to Theodore in regards to the moves her telecommunications company is making when she learns about it in their company's internal strategy meeting so he can make a better stock trade. This is known as ________. A) insider trading B) cramming C) embezzlement D) false token E) check kiting

A

Penny points a gun at Jose and threatens to shoot him. Jose is very brave and not frightened. The police arrive and Penny is arrested without shooting Jose. What tort did Penny commit? A) Assault only B) Battery only C) Assault and battery D) Assault, battery, and strict liability E) Penny's conduct does not constitute an assault because Jose was not afraid.

A

Restrictions on states' authority to pass laws that substantially affect interstate commerce are referenced by which of the following? A) The dormant commerce clause B) The superior commerce clause C) The interstate trafficking commerce clause D) The tracking clause E) The state authority clause

A

Ronaldo promised to sell Linda a Dalmatian puppy for $700 but backed out of the deal. Linda sued Ronaldo in state court for breach of contract and asked for a jury to hear the case. During jury selection, one juror, Mika, said that she did not think she could be fair to Linda because Linda did not appear to be a dog lover. Linda's lawyer asked that Mika not hear the case on the basis that she could not be fair, and the judge excused Mika. Linda also decided that another juror, Sandy, looked at her in a grumpy manner so, without a reason given, Linda's lawyer asked the judge to excuse that juror from serving. The judge did so. After the jury was chosen, Linda's lawyer made a statement to the jury, as did Ronaldo's lawyer. Linda's lawyer then called to the witness stand Jules, a friend of Linda, who heard the discussion held between Linda and Ronaldo regarding the purchase of the puppy. Jules testified under questioning by Linda's lawyer that she heard Linda say that she would pay $700 for the puppy and that she also heard Ronaldo say that he would sell the dog for that amount. Unfortunately for Linda, Jules also testified in response to questioning by Ronaldo's lawyer that Ronaldo distinctly told Linda that he would only sell the puppy to her if Linda came with cash for the puppy within seven days. Linda did not show up with the money for ten days and Ronaldo had already sold the dog to someone else. The judge ruled in favor of Ronaldo. Once Linda's attorney presented all the evidence, Ronaldo's attorney could ask for a[n] ________. A) directed verdict B) closing argument C) immediate dismissal D) redirect examination E) mistrial

A

Sam was illegally growing marijuana in his home using heat lamps. The police randomly selected his street to search for illegal activity. Without any kind of a warrant, they used a thermal imager, an instrument that detects heat emissions, and determined that given the amount of emissions coming from Sam's house, he must be engaged in criminal marijuana cultivation. The police immediately broke down Sam's door, searched his home, and arrested him. The police officer in charge told Sam that he had no right to stay silent and that he might as well confess. Sam proceeded to confess to growing marijuana and a number of other crimes. The judge was so angry with Sam that he threw him in jail for two months without any kind of hearing. Sam's lawyer, who just graduated from law school, is not sure if the police acted legally in Sam's case by using the thermal imager. Sam's lawyer also questions whether Sam had a right to remain silent, and he is concerned that the judge has not granted Sam a hearing. Was the officer correct in telling Sam he had no right to remain silent? A) The officer was incorrect, and Sam had the right to remain silent based on his constitutional protection against self-incrimination. B) The officer was correct that Sam had no right to remain silent, and there has never been a constitutional right entitling a defendant to remain silent. C) The officer was correct that Sam had no right to remain silent because while at one time there was a constitutional right entitling a defendant to remain silent, that right was repealed by constitutional amendment. D) The officer was correct, and Sam had no right to remain silent because the right against self-incrimination only protects against crimes involving physical harm to another. E) The officer was partially correct because while Sam had a right to remain silent about other crimes, he did not have the right to remain silent about the drugs found in the initial search.

A

Sana's idea is best referred to as ________. A) The Golden Rule B) The Disclosure Principle C) The Help Peers Test D) The Sarbanes-Oxley Rule E) The Greenhouse Rule

A

Sofia tells Gordon she wants to arbitrate their dispute. Gordon tells Sofia that if the arbitrator does not rule in her favor, they can appeal, but it is very difficult to overturn the decision of an arbitrator. Is Gordon correct? A) Yes, courts give extreme deference to arbitrators' decisions. B) No, arbitration decisions are final and binding. C) No, arbitration decisions are held to the same standard as a judge's ruling. D) No, arbitration decisions are often overturned because many arbitrators do not have a background in law. E) Yes, an arbitrator's decision is difficult to overturn because courts often do not have jurisdiction over a private arbitration.

A

Statements made by Ryan to his friends that were defamatory of Melissa are what type of defamation? A) Slander but not libel. B) Libel but not slander. C) Both libel and slander. D) No tort was committed because the statements were made only to friends of Ryan, not to business acquaintances of Melissa. E) No tort was committed because the falsehood involved matters of appearance, not business-related matters.

A

Tamra, a first year law student, slipped on a freshly mopped floor while walking to class. She bumped into Anton, another first year law student, breaking his glasses. He was very angry with Tamra and slashed all of her car tires causing damages of $1,500. Anton also decided to sue Tamra for negligence claiming damages of $300 for his broken glasses. He decided that he already knew all about the law and did not need a lawyer. Anton sued Tamra in state court. Tamra, in the same lawsuit, brought an action against Anton for slashing her tires. At the trial in state court, Tamra told the judge that a friend, Aimee, told Tamra that she saw Anton slash all of Tamra's tires. The judge disallowed Tamra's testimony on that issue. Aimee also came to court and testified about seeing Anton slashing all of Tamra's tires. The state court judge ruled in favor of Tamra awarding her $1,500 in damages. Anton said that he was not giving up and that he would seek double damages on appeal in federal court. Tamra and Anton live in different states when not attending school. After the trial, Tamra reported Anton's action of slashing her tires to the police, who said that they would proceed with a criminal action against Anton. Anton goes to see Alex, a recent graduate who had just passed the bar, and asked Alex to represent him in a federal court appeal. Tamra's action against Anton for the tires is called a ________. A) Counterclaim B) Cross-Claim C) Third-party claim D) Discovery claim E) Service claim

A

The 1998 ________ states that all federal district court litigants must consider the use of ADR at some stage of the litigation. A) Alternative Dispute Resolution Act B) Federal Court Early Intervention Act C) Dispute in Litigation Act D) Resolution Dispute Act E) Peacekeepers Act

A

The City of JohnPaul in the state of Illinois passed an ordinance that banned spray paint in the city as a means to reduce graffiti. Townsend Paint Suppliers, a huge spray paint provider from Michigan, sued the City of JohnPaul stating that the new ordinance violated the dormant commerce clause. Is Townsend Paint Suppliers correct? A) No, the legislation does not treat paint suppliers from out of state any differently than paint suppliers within the state and would be proper exercise of a state's police powers. B) No, the ordinance violates the First Amendment not the dormant commerce clause. C) No, this is a federal commerce clause violation and does not deal with any state interests. D) Yes, this is a violation of the dormant commerce clause. E) Yes, this is a violation but a federal court would most likely find that the commerce clause was violated instead of the dormant commerce clause.

A

The Federal Aviation Administration has banned the presence of giant lizards on airplanes. Corina has a business training lizards for use in television and movies. Her star lizard, a massive iguana named Izzy, likes to have her own seat on the airplane. In order to promote her business, Corina has asked her home state of Montana to pass a law permitting giant lizards on airplanes. 77) If Montana decided to modify it's state laws to permit giant lizards on airplanes, would the courts consider this action to be unconstitutional? A) Yes, due to the principals of federalism. B) No, because it would would infringe on states's rights. C) No, because Montana would modify the law. D) No, because full faith and credit would be implicated. E) Yes, under the equal protection clause.

A

The US District Courts are considered ________? A) trial courts B) appeals courts C) courts of last resort D) limited courts E) preemptory courts

A

The United States Code is where you can go to find ________. A) Laws enacted by Congress B) Presidential Enactments C) Laws of all 50 states D) Federal Supreme Court cases E) Court cases of all 50 states

A

The argument that traditional moral hierarchies ignore relationships is a part of which theory of business ethics? A) Ethics of care B) Absolutism C) Consequentialism D) Deontology E) Profit motive

A

The forceful and unlawful taking of personal property is which of the following? A) Robbery B) Burglary C) Larceny D) Arson E) Criminal fraud

A

The law in Hannah's state regulating using cell phones would be classified as which of the following? A) A statute B) A type of common law C) An executive order D) A uniform law E) A negative law

A

The president issued an executive order requiring that every citizen purchase a head of broccoli to eat once a week in order to maintain health. The executive order was challenged and the case went to the U.S. Supreme Court. The Supreme Court declared the executive order could not be enforced because it was an abuse of the president's power. What is the primary source of law the Supreme Court would interpret to determine if the executive order was enforceable? A) The Constitution B) State statutes C) Case law D) Uniform laws E) Executive orders

A

The supreme law of the land is the ________. A) U.S. Constitution B) Declaration of Independence C) U.S. Code D) Model Law E) Uniform Code

A

The term ________ or "guilty act" is used to describe the wrongful behavior in a criminal action. A) actus reus B) mens rea C) criminal actus D) locos actus E) carpe diem

A

Today, when the First Amendment is concerned, what does the term "political speech" reference with regard to corporations? A) Speech that occurs when corporations support political candidates. B) Speech by any citizen in connection with an election campaign. C) Speech by elected officials regarding other elected officials. D) Speech by private citizens involving advertising that is regulated by state government. E) While the term "political speech" has historical connections, the First Amendment is not referenced in relation to "political speech" today because today all speech is considered at the same level.

A

Tricking consumers into changing their phone service to another carrier without their consent is known as which of the following? A) Slamming B) Cramming C) Stuffing D) Fraudulent adding E) Fraudulent transfer

A

Under a conditional privilege, a party will not be held liable for defamation unless the false statement was made ________. A) with actual malice B) negligently C) in print D) publicly E) to an employer

A

Under the authority of the commerce clause, Congress can pass laws as long as what criteria are met? A) As long as the law affects commerce among the states, or interstate commerce, in some way, the regulation is generally constitutional. B) The law will only be constitutional if it affects taxation in some way. C) As long as the law involves, in some way, a right guaranteed by the Bill of Rights, it is constitutional. D) As long as the law is approved by any state affected, it is constitutional. E) As long as the law is not objected to by any state affected within six months of its passage, it is constitutional

A

Under what circumstances could government agents engage in a search without a search warrant? A) When law enforcement officials believe it is likely that the items sought will be removed before they can obtain a warrant. B) Outside of normal working hours for a judge. C) When they can show that the suspect who lives in the residence at issue has been in jail previously. D) When they can show that a felony is involved. E) When they can show that they have heightened probable cause.

A

Using fraudulent means to obtain information about someone's phone use is which of the following? A) Pretexting B) Ponzi scheme C) False pretenses D) False token E) Defalcation

A

What action would Jon have against Taylor for taking the rake? A) Trespass to personal property. B) Conversion. C) Private nuisance. D) Negligence. E) Harassment.

A

What did the court rule in regard to the plaintiff's claim of intentional infliction of emotional distress in Olson v. CenturyLink, the case in the text in which the plaintiff alleged that representatives of the defendant, a provider of telephone services, wrongly failed to process his application, disconnected his telephone service, and hung up on him during telephone conversations? A) That the conduct complained of by the plaintiff failed to meet the level of conduct required for a finding of the intentional infliction of emotional distress. B) That the plaintiff's complaint regarding the disconnection of his telephone service was sufficient to support a claim of intentional infliction of emotional distress but that the plaintiff's other allegations were insufficient to support an award based on the tort of the intentional infliction of emotional distress. C) That the plaintiff's complaint regarding the refusal to process his application was sufficient to support a claim of intentional infliction of emotional distress but that the plaintiff's other allegations were insufficient to support an award based on the tort of the intentional infliction of emotional distress. D) That the plaintiff's complaint regarding representatives hanging up on him were sufficient to support a claim of intentional infliction of emotional distress but that the plaintiff's other allegations were insufficient to support an award based on the tort of the intentional infliction of emotional distress. E) That the plaintiff would be allowed to proceed as to all allegations because they were all adequate to support a verdict based on the intentional infliction of emotional distress.

A

What do the letters "WH" mean in reference to the "WH Framework for Business Ethics" discussed in the text? A) Who and How B) When and How C) Why and Hope D) Who and Hope E) Where and How

A

What does protection against "double jeopardy" mean? A) The government cannot try a person more than once for the same crime. B) The government cannot try a person more than twice for the same crime. C) The government must have double proof to prosecute a person twice for the same crime. D) The government must show that at least two people were injured by the defendant in order to justify trying the defendant twice for the same crime. E) The government may impose a double sentence on a repeat offender in order to protect the public.

A

What does the free-exercise clause of the First Amendment provide? A) Government cannot make a law prohibiting choice in relation to religion. B) Government cannot make a law establishing a religion. C) Government cannot make a law referencing religion in any manner. D) Government must establish laws respecting religion in order to prevent disruptive behavior such as terrorism and to allow free-exercise by the majority. E) That only religions recognized when the First Amendment was passed are entitled to recognition and protection.

A

What does the value of security mean? A) To possess a large enough supply of goods and services to meet basic needs B) To possess the capacity or resources to act as one wishes C) To treat all humans identically, regardless of race, class, gender, age and sexual preference D) To possess anything that someone is willing to grant you E) To maximize the amount of wealth in society

A

What is a reason why the principle of deontology may be difficult to apply? A) People disagree about what duties we owe to one another and which duties are more important. B) People disagree about whether consequences are positive or negative. C) People disagree about the ethics of care. D) People disagree about what duties we owe to one another and also whether consequences are positive or negative. E) People disagree about what duties we owe to one another, whether consequences are positive or negative, and about the ethics of care.

A

What is the position of the Equal Employment Opportunity Commission (EEOC) on mediation? A) The EEOC encourages the mediation of employment discrimination claims. B) The EEOC discourages the mediation of employment discrimination claims. C) The EEOC discourages the mediation of employment discrimination claims in sexual harassment cases but not in age discrimination cases. D) The EEOC discourages the mediation of employment discrimination claims in age discrimination cases but not in sexual harassment cases. E) The EEOC encourages the mediation of all employment discrimination claims except for those arising under the Americans with Disabilities Act where the use of mediation is discouraged.

A

What is the role of the World Trade Organization? A) To stimulate negotiations among member nations and to adjudicate trade disagreements among member nations. B) To enforce U.S. trade laws globally throughout 160 member nations. C) To restrict the flow of goods across borders in order to promote and protect domestic trade among member nations. D) To promote standardized trade laws globally. E) To create model trade treaties such as NAFTA and GATT to help establish conditions of global trade.

A

What is the system of "guanxi" used in China? A) It refers to a system of relationship building woven together by social ties. B) It refers to a system of strict ethical rules. C) It refers to a prohibition against criticism of government rules and regulations. D) It refers to a system by which business people attempt to avoid strict Chinese regulations. E) It refers to a system of smuggling.

A

What is the term for a court order that authorizes law enforcement agents to search for or seize items specifically described in the order? A) Search warrant B) Subpoena warrant C) Search authorization form D) Seek warrant E) Review authorization

A

What is true regarding Ryan's defense that he thought he knew the law of Winland because he had studied it some years before and that the laws under which he was arrested were new? (Assume the law of Winland is the same as the U.S. on this issue.) A) Rules change and Ryan had a duty to stay current. It is not a defense that he studied the law at one time and that the laws at issue were new. B) Ryan is entitled to rely as a defense on the fact that he had studied the law at one time. Officials in Winland were charged with publicizing any changes. C) If Ryan can prove that the laws at issue were passed within the last twelve months before he was arrested, he will get out of jail because there is a twelve month grace period for new laws. D) If Ryan can prove that the laws at issue were passed within the last six months before he was arrested, he will get out of jail because there is a six month grace period for new laws. E) Ryan cannot be prosecuted so long as he was not aware he was violating the law. He did not have the requisite intent to perform a criminal act, regardless of whether the laws were new.

A

What standard of proof would the trial judge have applied when ruling on the case? A) A preponderance of the evidence standard B) A standard of proof beyond a reasonable doubt C) A standard of proof by a margin D) A standard of certainty in proof E) A standard of proof beyond a reasonable question

A

What was the Supreme Court ruling in Christy Brzonkala v. Antonio J. Morrison et. al., the case in the text involving the constitutionality of the section of the Violence Against Women Act providing for a civil remedy? A) That Congress lacked authority under the Commerce Clause to pass the section at issue. B) That because of its police powers, Congress had authority under the Commerce Clause to pass the section at issue. C) That because gender-motivated crimes were at issue, Congress had authority under the Commerce Clause to pass the section at issue. D) That because violent crime was involved, Congress had authority under the Commerce Clause to pass the section at issue. E) That because the petitioner established a link between the cost of crime against women and national productivity, Congress had authority under the Commerce Clause to pass the section at issue.

A

What was the result at the U.S. Supreme Court level in the Case Opener involving the constitutionality of the "individual mandate" of the Affordable Care Act requiring that most Americans obtain minimum essential health insurance contained within the Affordable Care Act? A) The Supreme Court upheld the "individual mandate" of the Affordable Care Act on the basis that it was a valid exercise of congressional power under the taxing clause. B) The Supreme Court upheld the "individual mandate" of the Affordable Care Act on the basis that it was a valid exercise of congressional power under the commerce clause. C) The Supreme Court upheld the "individual mandate" of the Affordable Care Act on the basis that it was a valid exercise of congressional power under the taxing clause and also under the commerce clause. D) The Supreme Court struck the "individual mandate" from the Affordable Care Act as a violation of the commerce clause but allowed the remainder of the law to remain. E) The Supreme Court struck the "individual mandate" from the Affordable Care Act as a violation of the taxing clause but allowed the remainder of the law to remain.

A

What, if any, defenses are available to a person who has been accused of defamation? A) Truth and privilege. B) Privilege but not truth. C) Truth but not privilege. D) Purposeful and negligence. E) There are no defenses to defamation.

A

Which areas of business law would a human resources manager find most relevant? A) Contracts, employment and labor law, and employment discrimination B) Contracts only C) Contracts and labor law, but not employment discrimination D) Employment and labor law, but not contracts E) Human resource managers hold administrative positions and are not concerned with areas of business law

A

Which is a true statement regarding actions taken by news corporations to protect confidential records? A) News corporations are putting numerous security barriers and encryptions around their records so that nobody can gain access to them. B) News corporations are attempting to protect information written by their own employees, but efforts are not made to protect confidential sources because constitutional protection does not extend to confidential sources. C) News corporations are not bothering with security barriers to the extent as in previous decades because government regulation requires that information be turned over, and there is no point in taking drastic measures to protect such information. D) New regulations make significant privacy barriers illegal in regard to news corporations, so many barriers have been or are being removed. E) Because of the respect given to confidential records by the U.S. government and its enforcement agencies, news corporations see little need to enhance protection of confidential records.

A

Which of the following are typically classified white-collar crime? A) Mail fraud, bribery, embezzlement, and computer crimes. B) Mail fraud, larceny, burglary, robbery, and arson. C) Mail fraud, bribery, larceny, embezzlement, and computer crimes. D) Bribery, embezzlement, larceny, and computer crimes. E) Bribery, embezzlement, burglary, and computer crimes.

A

Which of the following cases overturned prior precedent? A) Brown v. Board of Education B) Wygant v. Jackson Board of Education C) Planned Parenthood of Southeastern Pennsylvania v. Casey D) Plessy v. Ferguson E) Precedent cannot be overturned

A

Which of the following holds that a person may be deemed not guilty by reason of insanity, even if he or she knew that a criminal act was wrong, as long as the defendant was driven to commit the crime from some urge resulting from a mental deficiency the defendant could not resist? A) The irresistible impulse test. B) The involuntary act. C) The substantial capacity test. D) The duress defense. E) The necessity defense.

A

Which of the following involves making a false statement about a business product or service that results in a loss of sales? A) Disparagement. B) Interference with a contract. C) Conversion. D) Derogation. E) Abuse of title.

A

Which of the following is a court's power to hear certain kinds of cases? A) Subject-matter jurisdiction B) In kind jurisdiction C) In personam jurisdiction D) In loco jurisdiction E) In area jurisdiction

A

Which of the following is a path that a case could take through the U.S. court system? A) State trial court to a state supreme court to the U.S. Supreme Court. B) State trial court to the U.S. Supreme Court. C) U.S. district court to the state supreme court to the U.S. Supreme Court. D) State trial court to the U.S. district court to the U.S. Supreme Court. E) State trial court to the U.S. circuit court of appeals.

A

Which of the following is a question of fact? A) Did the vehicle run the traffic light? B) Is premeditation necessary for a first degree murder conviction? C) Is speech protected by the First Amendment? D) What is necessary for service of process? E) What elements should an answer contain?

A

Which of the following is a term describing the wrongful state of mind needed for a criminal action? A) Mens rea. B) Actus reus. C) Res judicata. D) Stare decisis. E) Carpe diem.

A

Which of the following is accurate regarding the speed and cost of ADR as compared to litigation? A) It is usually faster and cheaper. B) It is usually faster but more expensive. C) It is usually slower and more expensive. D) It is usually slower but cheaper. E) No studies have been conducted, so issues regarding the speed and cost of ADR as opposed to traditional litigation is unknown.

A

Which of the following is an example of how ethics shapes business laws? A) Contract laws require parties to live up to promises because keeping promises is part of our ethical inclination to encourage trust. B) Bribery is legal for U.S. companies doing business in foreign countries because respecting foreign cultures is part of our community ethics. C) Business law requires companies to put profits above all else, because protecting the rights of shareholders is the ethically proper course of action. D) Contract laws require parties to obey even unenforceable agreements because of our ethical belief that every promise must be kept at all times. E) Bribery is legal for U.S. companies doing business in foreign countries because foreign businesses can pay bribes and our ethical inclination is to level the playing field for American businesses.

A

Which of the following is an executive agency? A) The Federal Deposit Insurance Corporation B) The Federal Trade Commission C) The Securities and Exchange Commission D) The Federal Communications Commission E) The Nuclear Regulatory Commission

A

Which of the following is asserted by the principle of rights? A) That whether a business decision is ethical depends on how the decision affects the rights of all involved. B) That whether a business decision is ethical depends on how the decision affects the rights of stockholders without consideration of other stakeholders. C) That whether a business decision is ethical depends on how the decision affects the rights of employees without consideration of other stakeholders. D) That whether a business decision is ethical depends on how the decision affects the overall economy. E) That whether a business decision is ethical depends on how the decision affects the rights of organized labor without consideration of other stakeholders

A

Which of the following is recognized by the Chinese constitution or laws? A) Citizens' rights. B) Natural rights. C) Freedom of assembly. D) Freedom of speech. E) Human rights.

A

Which of the following is reported in the United States Code? A) Statutes B) Executive orders C) Common law D) Administrative law E) Local ordinances

A

Which of the following is the making of an entry into the books of a bank or corporation that is designed to represent the existence of funds that do not exist? A) False entry B) Defalcation C) False pretenses D) Ponzi scheme E) Pretexting

A

Which of the following is true regarding an arrest without a warrant? A) A law enforcement agent can arrest a suspect without a warrant if the officer believes there is probable cause but not enough time to obtain a warrant. B) A law enforcement officer may not arrest a suspect without a warrant even if the officer believes there is probable cause but insufficient time to obtain a warrant. C) A law enforcement agent can arrest a suspect without a warrant only if a felony is involved. D) A law enforcement officer can arrest a suspect without a warrant only if a violent crime is involved. E) A law enforcement officer can arrest a suspect without a warrant only if the suspect is a repeat offender.

A

Which of the following is true regarding areas in which arbitration is used? A) Arbitration is used in employment, medical malpractice cases, environmental disputes, commercial contract disputes, and insurance liability claims. B) Arbitration is used in medical malpractice cases, environmental disputes, and commercial contract disputes, but not in insurance liability claim disputes because those claims are governed strictly by state regulatory law. C) Arbitration is used in environmental disputes, commercial contract disputes, and insurance liability claim disputes, but not in medical malpractice cases because those cases are governed strictly by state statutory law and also state common law on negligence. D) Arbitration is used in medical malpractice cases, environmental disputes, and insurance liability claim disputes, but not in commercial contract disputes because those disputes are governed by the UCC. E) Arbitration is used in medical malpractice cases, commercial contract disputes, and insurance liability claim disputes, but not in environmental disputes because those disputes are governed strictly by federal law.

A

Which of the following is true regarding business ethics? A) Business ethics refers to standards of business conduct but does not result in a set of correct decisions. B) Business ethics results in a set of correct decisions and does not simply refer to standards of business conduct. C) Business ethics produces a list of correct business decisions that all ethical businesses will make so long as the theory of deontology is followed. D) Business ethics produces a list of correct business decisions that all ethical businesses will make so long as the theory of rule utilitarianism is followed. E) Business ethics produces a list of correct business decisions that all ethical businesses will make so long as the theory of virtue ethics is followed

A

Which of the following is true regarding the protection of "fighting words" under the First Amendment? A) Fighting words are unprotected speech under the First Amendment. B) Fighting words are protected speech under the First Amendment only if they are uttered by an individual citizen, not by a corporate representative. C) Fighting words are protected speech under the First Amendment only if they involve political activity. D) Fighting words are protected speech under the First Amendment only if they are made in connection with a protest march. E) Fighting words are protected speech under the First Amendment only if they are made in conjunction with self-defense.

A

Which of the following is true regarding treaties at the federal level? A) A treaty is generally negotiated by the executive branch and must then be approved by two-thirds of the Senate. B) A treaty is generally negotiated by the executive branch and must then be approved by two-thirds of the House of Representatives. C) A treaty is generally negotiated by the executive branch and must then be approved by two-thirds of the Senate and also by two-thirds of the House of Representatives. D) A treaty is generally negotiated by the executive branch and no approval by the Senate or House of Representatives is needed. E) A treaty is generally negotiated by the executive branch and must then be approved by two-thirds of the Senate and also by two-thirds of the state legislatures.

A

Which of the following requires a "pattern"? A) The Racketeer Influenced and Corrupt Organizations Act. B) The False Claims Act. C) The Sarbanes-Oxley Act of 2002. D) The crime of false pretenses. E) The crime of false token.

A

Which of the following torts seek to compensate people wrongfully subjected to litigation? A) Malicious prosecution, wrongful civil proceedings, and abuse of process. B) Wrongful civil proceedings, but not malicious prosecution or abuse of process. C) Abuse of process, but not malicious prosecution or wrongful civil proceedings. D) Malicious prosecution and wrongful civil proceedings, but not abuse of process. E) Malicious prosecution and abuse of process, but not wrongful civil proceedings.

A

Which of the following would be involved in the prosecution of Ashley for slapping Bob? A) Public law and criminal law B) Private law and criminal law C) Private law and civil law D) Public law and civil law E) Civil law and criminal law

A

Which of the following would constitute false imprisonment? A) Threatening to use immediate physical force to detain someone; refusing to release property; or physical restraint, such as tying someone to a chair; but not moral pressure. B) Threatening to use immediate physical force to detain someone; physical restraint, such as tying someone to a chair; or moral pressure; but not refusing to release property. C) Physical restraint, such as tying someone to a chair; but not threatening to use immediate physical force to detain someone; refusing to release property; or moral pressure. D) Threatening to use immediate physical force to detain someone; refusing to release property; physical restraint, such as tying someone to a chair; and moral pressure. E) Physical restraint; threatening to use immediate physical force; but not refusing to release property; or moral pressure.

A

Which of the following would constitute the privacy tort of false light? A) Attributing characteristics or beliefs to a person that he or she does not possess. B) Disclosing private facts about a person in order to obtain some sort of personal gain. C) Defaming a person with actual malice. D) Defaming a public figure. E) Defaming a public figure for personal gain.

A

Which of these apply to the defamation printed in the school newspaper? A) It is libel but not slander. B) It is slander but not libel. C) It is both libel and slander. D) No tort was committed because an editorial, not a formal news report, was involved. E) No tort was committed because the falsehood involved matters of appearance, not business-related matters.

A

Which statement is true concerning the relationship between ethical relativism and situational ethics? A) Like ethical relativism, situational ethics requires that we evaluate the morality of an action by imagining ourselves in the position of the person facing the ethical dilemma; but unlike ethical relativism, situational ethics allows us to judge other people's actions. B) Like ethical relativism, situational ethics requires that we evaluate the morality of an action by imagining ourselves in the position of the person facing the ethical dilemma and also like ethical relativism, situational ethics allows us to judge the actions of others. C) While the two theories are similar, ethical relativism upholds the existence of objective moral standards whereas situational ethics refuses to recognize the existence of objective moral standards. D) Ethical relativism and situational ethics are the same theory. E) There is no relationship between ethical relativism and situational ethics because the theories are fundamentally opposed to each other.

A

Which statement is true regarding meeting the minimal standard of business ethics suggested by Jessica? A) Decisions must be legal. B) Decisions must meet the criteria of a follower of deontology. C) Decisions must meet the criteria of a follower of utilitarianism. D) Decisions must receive a majority vote of acceptance by employees. E) Decisions must be legal and decisions must also receive a majority vote of acceptance by employees.

A

Which statement is true regarding the intent required for an intentional tort? A) The intent at issue is not intent to harm but, rather, is intent to engage in a specific act, which ultimately results in a physical or economic injury to another. B) The intent at issue is not intent to harm but, rather, is intent to engage in a specific act, which ultimately results in a physical injury to another. Economic injury is insufficient. C) The intent at issue is intent to harm that results in a physical or economic injury to another. D) The intent at issue is intent to harm that results in physical injury to another. Economic injury is insufficient. E) The intent at issue is not intent to harm and is not intent to engage in a specific act. Negligence will suffice.

A

Which statement is true regarding tort litigation? A) Tort litigation has been declining gradually since 1990. B) Tort litigation has increased steadily since 1990. C) Tort litigation has remained constant with no real change in numbers since 1990. D) Tort litigation increased significantly in the 1990s but has declined in the last two years. E) Tort litigation decreased in the 1990's but has increased steadily since the year 2000.

A

Which statements are true regarding Ashley's claim against Bob for damages involving the laptop and her claim that Bob maliciously had her arrested? A) The claim for damages involving the laptop and also the claim that Bob maliciously had her arrested are civil claims involving private law. B) The claim for damages involving the laptop is a civil claim involving private law, but the claim that Bob maliciously had her arrested is a civil claim involving public law. C) The claim for damages involving the laptop is a civil claim involving private law, but the claim that Bob maliciously had her arrested is a criminal claim involving public law. D) The claim for damages involving the laptop is a civil claim involving private law, but the claim that Bob maliciously had her arrested is a criminal claim involving private law. E) The claim for damages involving the laptop and also the claim that Bob maliciously had her arrested are criminal claims involving private law.

A

Why should business managers be familiar with laws that regulate business practices between nations? A) Advances in trade and technology have made trade with other countries easy, but companies must comply with local laws. B) NAFTA requires registered businesses to understand and comply with international law. C) GATT mandates that U.S. organizations know the laws in the United States and in 150 other member countries in order to incorporate. D) Businesses are required to know the laws and to become WTO members to have Internet websites. E) Businesses may meet foreigners and should be familiar with their customs to foster positive business relationships

A

Will a plaintiff be allowed to assert jurisdiction over a defendant in the plaintiff's state for a cause of action arising out of the defendant's website? A) It depends on the nature and quality of commercial activity that an entity conducts over the Internet. B) Yes, for any type of action. C) Yes, but only if the defendant consented to jurisdiction in the plaintiff's home state. D) Yes, but only if the defendant has actually physically been in the plaintiff's home state within the 180 days prior to the filing of the complaint. E) No, not under any circumstances.

A

Written questions that one party sends to another to answer under oath are called ________. A) Interrogatories B) Depositions C) Inquiries D) Subpoenas E) Sworn assertions

A

________ damages are awarded to punish the defendant and to deter others from similar conduct. A) Punitive B) Compensatory C) Nominal D) Retaliatory E) Revenge

A

________ law cases involve either two individuals or two organizations. A) Civil B) Legal realism C) Criminal D) Precedent E) Public

A

________ would see murder in the realm of self-defense as a moral action as long as the murderer believes that the action is justified for his self-preservation. A) Ethical relativism B) Deontology C) Categorical imperative D) Absolutism E) Kantian

A

_______damages are a small amount of money given to recognize that a defendant did indeed commit a tort in a case in which there were no compensable damages suffered by the plaintiff. A) Nominal B) Compensatory C) Punitive D) Retaliatory E) Revenge

A

Ethical conversation is primarily about finding the one and only right thing to do.

Answer: FALSE

Ronaldo promised to sell Linda a Dalmatian puppy for $700 but backed out of the deal. Linda sued Ronaldo in state court for breach of contract and asked for a jury to hear the case. During jury selection, one juror, Mika, said that she did not think she could be fair to Linda because Linda did not appear to be a dog lover. Linda's lawyer asked that Mika not hear the case on the basis that she could not be fair, and the judge excused Mika. Linda also decided that another juror, Sandy, looked at her in a grumpy manner so, without a reason given, Linda's lawyer asked the judge to excuse that juror from serving. The judge did so. After the jury was chosen, Linda's lawyer made a statement to the jury, as did Ronaldo's lawyer. Linda's lawyer then called to the witness stand Jules, a friend of Linda, who heard the discussion held between Linda and Ronaldo regarding the purchase of the puppy. Jules testified under questioning by Linda's lawyer that she heard Linda say that she would pay $700 for the puppy and that she also heard Ronaldo say that he would sell the dog for that amount. Unfortunately for Linda, Jules also testified in response to questioning by Ronaldo's lawyer that Ronaldo distinctly told Linda that he would only sell the puppy to her if Linda came with cash for the puppy within seven days. Linda did not show up with the money for ten days and Ronaldo had already sold the dog to someone else. The judge ruled in favor of Ronaldo. 78) When lawyers choose a jury, it is called ________. A) Voir dire B) Jury analysis C) Jury review D) Ven dere E) Shadowing

A; Once the potential jurors have reported for jury duty, the voir dire, or jury selection, process begins

Concurrent federal jurisdiction means that both state and federal courts have jurisdiction over a case.

Answer: TRUE

ADR is a process done only in the U.S., not in foreign countries.

Answer: FALSE

According to the case United States v. Park, corporate executives cannot be held accountable for crimes due to their failure to act.

Answer: FALSE

Any person can bring a case to court if they believe they have been harmed.

Answer: FALSE

Arbitration is the primary ADR process used in federal district courts.

Answer: FALSE

Assault and battery are always claimed at the same time.

Answer: FALSE

Because of double jeopardy, a person found not guilty of a criminal offense cannot also face a civil charge for the same alleged actions.

Answer: FALSE

Business ethics and regular ethics are the exact same.

Answer: FALSE

China punishes embezzlement and bank fraud less severely than the U.S.

Answer: FALSE

Consequentialism provides a rigid set of rules to follow regardless of the situation.

Answer: FALSE

Constitutions and statutes always cover all of the detailed rules that affect business and government relations.

Answer: FALSE

Corporations are not protected by the First Amendment.

Answer: FALSE

Courts are generally critical and unsupportive of ADR methods.

Answer: FALSE

Criminal law lays out the rights and responsibilities implied in relationships between persons.

Answer: FALSE

Currently there are only a few countries belonging to the United Nations Convention on the Recognition of Enforcement of Foreign Arbitral Awards.

Answer: FALSE

Defense of property cannot be a defense to a claim of battery.

Answer: FALSE

If a law prevents individuals from exercising a fundamental right, or if the law's classification scheme involves suspect classifications, such as race, national origin or citizenship, the action will be subject to the rational-basis test.

Answer: FALSE

If a potential juror's response to a question indicates bias, an attorney should use a peremptory challenge to remove the juror.

Answer: FALSE

In an ethical analysis using the WH Framework referenced in the text, owners are the most important stakeholders and should receive the greatest consideration in decision making regardless of the type of problem addressed.

Answer: FALSE

In order to perform arbitration, a person must be a lawyer.

Answer: FALSE

Insurer American International Group (AIG) is well known in the insurance industry for its ethical business practices.

Answer: FALSE

Judicial review is explicitly laid out in the Constitution.

Answer: FALSE

Juries decide questions of law.

Answer: FALSE

Mediators are always retired judges who can mediate based on their experiences on the bench.

Answer: FALSE

Model or uniform laws are a source of law that are binding on courts.

Answer: FALSE

Most states define larceny as stealing the property of another and converting it to your own personal use.

Answer: FALSE

Negotiation in which each side seeks to maximize its own gain is called problem-solving negotiation.

Answer: FALSE

Negotiation is a bargaining process in which disputing parties interact informally, but only with lawyers, to attempt to resolve their dispute.

Answer: FALSE

Only civil law regulates the behavior of individuals.

Answer: FALSE

Only one party may appeal from a final judgment.

Answer: FALSE

Private companies do not use mediation to resolve internal disputes with employees.

Answer: FALSE

Public law controls both disputes between private individuals and their government and disputes involving only private individuals.

Answer: FALSE

Statutory law is the supreme law of the land and the foundation for all laws in the United States.

Answer: FALSE

Subjective opinions are subject to an action for defamation in the same way as statements of alleged fact.

Answer: FALSE

The Third Amendment protects citizens from unreasonable search and seizure.

Answer: FALSE

The US legal system has one main court system.

Answer: FALSE

The commerce clause does not allow Congress to regulate commerce with foreign Nations, only the President can do that.

Answer: FALSE

The community in which a firm operates would not be considered a stakeholder of the firm.

Answer: FALSE

The resolution of legal disputes through methods other than litigation is called CAR.

Answer: FALSE

The right to privacy is expressly provided for in the Constitution.

Answer: FALSE

The system of checks and balances on the federal government does not include a check on the judicial branch because they determine what is constitutional.

Answer: FALSE

The term "Bill of Rights" includes all amendments passed in support of the Constitution.

Answer: FALSE

The terms shareholder and stakeholder have the same meaning.

Answer: FALSE

Tort law is set by federal law that is then implemented by the states.

Answer: FALSE

Torts can be classified into three classifications in the U.S.: intentional, accidental and negligence.

Answer: FALSE

Under federal statutory law, Internet transactions cannot ever be the basis for a finding of in personam jurisdiction.

Answer: FALSE

When courts overturn precedent and create new precedent, they are obeying the principle of stare decisis.

Answer: FALSE

White-collar crime is not punishable by imprisonment under our existing criminal laws.

Answer: FALSE

Under the WH framework the "W" stands for Who and the "H" stands for help.

Answer: FALSE; who and how

A corporation cannot be considered a criminal under common law when mens rea is required because it is not an actual person and, thus, does not have a mind.

Answer: TRUE

A defendant who believes that he or she has a claim against the plaintiff would include a counterclaim with the answer.

Answer: TRUE

An arbitrator is more likely to issue a compromise decision than a judge.

Answer: TRUE

Another name for case law is common law.

Answer: TRUE

Business ethics is the application of ethics to the special problems and opportunities experienced by business people.

Answer: TRUE

Business law consists of the enforceable rules of conduct that govern commercial relationships

Answer: TRUE

By broadening the scope of actions covered and the definition of a public official, Congress amended the Foreign Corrupt Practices Act to conform to the antibribery convention adopted by the Organization for Economic Cooperation and Development.

Answer: TRUE

Case law interpretations are law, unless they are revoked later by new statutory law.

Answer: TRUE

Ethical relativism is a theory of ethics that denies the existence of objective moral standards and teaches that individuals must evaluate actions on the basis of what they feel is best for themselves.

Answer: TRUE

Government inspectors may enter a business to conduct an inspection without a warrant if the industry is highly regulated such as food or firearms.

Answer: TRUE

If a court awards $1 million to compensate an injured plaintiff for medical bills, loss of wages, and pain and suffering, this is an example of compensatory damages.

Answer: TRUE

If a defamation statement was originally broadcast by a company in the United States and was rebroadcast in the United Kingdom without the consent of the originator of the broadcast, the U.S. company may still be held liable in the United Kingdom court.

Answer: TRUE

If mediation is unsuccessful, the parties can turn to litigation, however, nothing said during the mediation can be used in the litigation because the mediation process is confidential.

Answer: TRUE

If you are a party to a dispute and determining which dispute method is best, you may want to ask if you want to keep the dispute private.

Answer: TRUE

In Berghuis v. Thompkins, the majority of the U.S. Supreme Court ruled that absent an explicit invocation of the right to remain silent, police may continue to interrogate a suspect, and what the suspect says may be used against him in court.

Answer: TRUE

In order to demonstrate a claim under the Racketeer Influenced and Corrupt Organizations (RICO) Act, one must show proof of more than one wrongful act.

Answer: TRUE

In rem jurisdiction is jurisdiction over property.

Answer: TRUE

In some cases, the U.S. Supreme Court functions as a trial court of limited jurisdiction.

Answer: TRUE

In some countries, businesses must pay bribes to receive supplies they need to stay in business.

Answer: TRUE

Law and business ethics serve as an interactive system that informs and assesses each other.

Answer: TRUE

Long arm statutes require minimum contacts to serve out-of-state defendants.

Answer: TRUE

Nominal damages are trivial damages intended to recognize that a defendant committed a tort against the plaintiff.

Answer: TRUE

Not all corporate speech is political speech.

Answer: TRUE

One form of consequentialism is utilitarianism.

Answer: TRUE

One type of ADR method is med-arb.

Answer: TRUE

Presidents base the power to issue executive orders on Article II, Section 3 of the Constitution, which establishes the constitutional power of the president to "take care that the laws be faithfully executed."

Answer: TRUE

Proving that consent occurred is a defense to battery.

Answer: TRUE

Reasonable apprehension of fear is the test for whether an assault occurred.

Answer: TRUE

Reinforcing a vision of a just society is one of the purposes of tort law.

Answer: TRUE

Social responsibility of business deals with the expectations that a community would impose on a firm doing business in their community.

Answer: TRUE

Some other countries' constitutions provide rights that American citizens do not have.

Answer: TRUE

Subject-matter jurisdiction is a court's power to hear certain kinds of cases.

Answer: TRUE

The Foreign Corrupt Practices Act makes it unlawful for a U.S. person to make payments to a foreign official to obtain or retain business with any person.

Answer: TRUE

The U.S. Constitution establishes a system of government based on the principle of federalism.

Answer: TRUE

The decision of a state supreme court is binding on a lower court that is located in the same state.

Answer: TRUE

The landmark early U.S. Supreme Court case of Marbury v. Madison recognized the right of courts to exercise judicial review in order to determine the constitutionality of laws.

Answer: TRUE

The primary purpose of criminal law is to punish an offender for causing harm to the public health, safety, or morals.

Answer: TRUE

The privileges and immunities clause prohibits states from discriminating against citizens of other states when those nonresidents engage in ordinary and essential activities.

Answer: TRUE

There are many purposes of the law. One such purpose is serving as a moral guide by indicating minimal expectations of citizens and organizations.

Answer: TRUE

To minimize extra costs, businesses will sometimes include in their contracts a forum selection clause.

Answer: TRUE

Usually the issue of ripeness arises when one party claims that a case is moot.

Answer: TRUE

When only a prospective contract exists, simply offering a better deal is not enough to create liability for intentional interference with contract.

Answer: TRUE

White-collar crime is defined as a variety of nonviolent illegal acts against society that most often occur in the business context.

Answer: TRUE

Big City also passed a regulation prohibiting any new construction on nearby private land in order to protect wildlife near the new shopping mall. Ivan had intended to build a house on his private property, but, because of the new regulation, his property is useless to him. Ivan meets with Yuri, an attorney, who says that Ivan is entitled to compensation for the government's actions. Is Yuri correct? A) No, because Ivan still owns the land. B) Yes, but only if Ivan is deprived of all economically beneficial uses of the land. C) No, because eminent domain does not apply. D) No, because the commerce clause allows him to sell the property. E) Yes, because the government's actions violate the equal protection clause.

B

Many federal district courts use ADR to resolve disputes within cases. Which form of ADR is most often used? A) Arbitration B) Mediation C) Federal summation D) Federal private trials E) Minitrials

B

A defendant who argues that commission of a crime was necessary to prevent a more severe crime from occurring is relying upon the defense of ________. A) Entrapment B) Necessity C) Justifiable use of force D) Duress E) Inescapable action

B

A difficulty with the application of virtue ethics is ________. A) The applicable categorical imperatives B) The lack of agreement about the meaning of "the good life" C) The applicable principle of rights D) The ethics of care analysis E) The lack of agreement regarding which rules on balance produce the greatest amount of pleasure for the greatest number of people

B

A person unlawfully enters a building with the intent to commit a felony, this would be considered a(n) ________. A) robbery B) burglary C) larceny D) arson E) intentional crime

B

According to our system of checks and balances, what can the other branches of government do if the President vetoes a bill that Congress believes is necessary to protect the welfare of the people? A) The Senate can overturn the veto with a two-thirds vote. B) Congress can overturn the veto with a two-thirds vote. C) The Senate can overturn the veto with a three-fifths vote. D) The Supreme Court can overturn the veto. E) The Supreme Court can declare the bill unconstitutional.

B

Adriana Gomez is a newly hired Production Manager for ABC Corporation which manufactures wind turbines. As a new manager, ABC Corp sends Adriana to training to familiarize herself with laws that might impact her work. What laws would most likely impact a production manager? A) White collar crime, contracts, antitrust laws B) Tort law, environmental law, contracts C) Antitrust law, contracts, intellectual property law D) Property law, negotiable instruments, contracts E) Consumer law, international law, employment law

B

Aiden, who just started at his new job as an assistant manager at a fast-food restaurant, wants to make a good impression. He thinks that things are going well. On only the second day on the job, however, he sees his manager, Debbie, slink out of the restaurant with a big box of hamburgers. She puts them in her car and speeds away. Aiden is fairly certain that Debbie did not pay for the hamburgers. Aiden asks advice of his best friend, Kyle; his girlfriend, Mia; his sister, Katie; and his dad, Joe. Kyle says that there are no real objective standards and that Aiden should just decide to do what is best for himself. Mia says that Aiden should think about the consequences and focus on the greatest benefit to all involved. She believes that it will do no real harm for Aiden to keep his mouth shut because the fast-food restaurant is making plenty of money and probably does not pay Debbie enough anyway. Plus, getting in trouble with Debbie could cause problems at work, and if Aiden loses his job, Aiden and Mia might have to put off their impending marriage. Katie says that on balance, following the rule of producing the greatest good would mean that he should report Debbie to her boss because stealing does not generally produce the greatest satisfaction. Aiden's dad says that as a matter of principle, Aiden should report Debbie to her boss because stealing is simply wrong. 68) Which theory most accurately fits the advice given by Mia? A) Deontology B) Act utilitarianism C) Rule utilitarianism D) Ethical relativism E) Virtue ethics

B

An assault victim sues the person who assaulted her. The court awards her $10,000 to pay for her actual medical expenses, to compensate her for time missed from work, and to compensate her for her pain. The $10,000 is an example of what type of damages? A) Nominal B) Compensatory C) Punitive D) Retaliatory E) Revenge

B

An ethical theory that urges managers to take those actions that provide the greatest pleasure after having subtracted the pain or harm associated with the action in question is called ________. A) Deontology B) Utilitarianism C) Kantian ethics D) Absolutism E) Ethical relativism

B

Billy witnessed a hit-and-run but is gravely ill with cancer. Xavier, who was injured in the accident, would like to preserve Billy's testimony for trial in case Billy dies before the trial date. What should Xavier do? A) Send interrogatories to Billy. B) Take Billy's deposition. C) Send a request to admit to Billy that the accident was the defendant's fault. D) Have a conference with the judge and Billy. E) There is nothing Xavier can do.

B

By buying stock, Claire is guilty of which offense, if any? A) She is guilty of insider trading because she tipped information. B) She is guilty of an insider-trading violation because she traded on tipped information. C) She is guilty of extortion. D) She is guilty of larceny. E) She is not guilty of any offense.

B

Delilah worked as a receptionist for Wild Music, a talent agency with many celebrity clients. Delilah also had her own business selling high-end skin care products, which she ran out of her studio apartment. She often contacted skin care clients through her work e-mail, as it was very convenient to search through Wild Music's confidential client list to generate bulk e-mails. 83) What crime, if any, has Delilah committed? A) Cyber terrorism B) Theft of company assets C) Hacking D) Extortion E) Larceny

B

Diego's company is looking to expand its business in the United States. His supervisor asks him what the company should consider from a legal perspective while evaluating which state to expand the business to. Which of the following should Diego consider? A) Companies should consider the tax laws and infrastructure for an expanding company, not the legal system since each state's legal system models the federal laws. B) Companies should look at the laws of the state, the court system located in the state, and whether the state's court system seems hospitable to business. C) Companies should evaluate how many businesses have succeeded in lawsuits within the state to determine if the state's judges are pro-business. D) Legally speaking, all state court systems follow the same Model Rules for Business and should not treat companies expanding into their state any differently. E) Companies should look at the success rate of lawyers defending companies that specialize in business litigation in each state to determine if the laws and court system seem fair to business.

B

Does the Sarbanes-Oxley act create a board of oversight; and, if so, what is it called? A) Chris is correct. No oversight board was created by the act. B) Yes, a board called the Public Company Accounting Oversight Board was created by the act. C) Yes, a board called the Public Accountability Commission was created by the act. D) Yes, a board called the CPA Oversight Commission was created by the act. E) Yes, a board called the Federal Accountability Commission was created by the act.

B

Emilia filed a charge of discrimination with the Equal Employment Opportunity Commission (EEOC), alleging that her employer, MegaTron, failed to promote her because of her nationality and gender. The EEOC had already received several discrimination charges from other employees alleging similar conduct by MegaTron. When Emilia was hired, she signed an employment contract with a clause stating that all work-related claims must be resolved through arbitration. 74) If, after filing her charge of discrimination with the EEOC, Emilia then filed a discrimination lawsuit against MegaTron, what would the court most likely do? A) Dismiss the lawsuit, because of Emilia's poor performance. B) Dismiss the lawsuit because of the binding arbitration clause in her employment contract. C) Allow the lawsuit to move forward because employees have a right to file a lawsuit. D) Allow the lawsuit because Emilia did not foresee she might suffer discrimination. E) Allow the lawsuit because arbitration clauses in employment contracts are not binding on employees.

B

Fajiha is a billing expert for a healthcare facility that treats individuals who are covered only by Medicare and not private health insurance. Fajiha learns that her employer is committing fraud against the government by filing false Medicare claims. Under which of the following might she obtain a portion of any recovery obtained by the government? A) The Medicare False Claims Act. B) The False Claims Act. C) The Sarbanes-Oxley Act of 2002. D) The Medicare Recovery Act. E) The Healthcare Fraud Act.

B

How many amendments are included in the Bill of Rights in the U.S. Constitution. A) Eight B) Ten C) Thirteen D) Twenty E) Twenty-five

B

If a person uses an explosive device in order to destroy a competitor's building, this is known as ________. A) a white-collar crime B) arson C) larceny D) burglary E) intentional destruction

B

If after reviewing the evidence in the case, there is no factual dispute and one party is entitled to judgment prior to trial, the court may grant which of the following? A) Motion for judgment on the pleadings B) Motion for summary judgment C) Motion for sanctions D) Motion for discovery E) Motion for production

B

In a criminal case against Anton, what standard of proof would the judge apply? A) A preponderance of the evidence standard B) A standard of proof beyond a reasonable doubt C) A standard of proof by a margin D) A standard of certainty in proof E) A standard of proof beyond a reasonable question

B

In a much heated debate on the Senate floor in regards to whether or not to allow a new legislative reform to be voted on, two senators call each other liars, cheats and other unflattering words. One senator also accuses the other one of bribery. If one of the senators sues the other for defamation, is there a defense available to the defamation claim? A) Yes, a limited privilege B) Yes, an absolute privilege C) Yes, a legal privilege D) Yes, a negligence privilege E) No, there is no privilege allowed for defamation.

B

In the past 20 years there have been major accounting scandals that have impacted many companies resulting in the illustration that the business world cannot be allowed to regulate itself ethically and produced the Sarbanes-Oxley Act. Who were two of the major violators? A) Ford Motor Company and WorldCom B) WorldCom and Enron C) Shell Corporation and Amazon D) Enron and the SEC E) Chrysler Corporation and WorldCom

B

In what state is XYZ Bank, a federally chartered bank, considered a citizen? A) The bank is considered a citizen of any state in which it is located, meaning any state in which the bank has branches. B) The bank is considered a citizen of the state in which it is located, meaning the state in which its main office, as specified in its articles of incorporation, is located. C) The bank is considered a citizen only of Washington, D.C. D) The bank is considered a citizen only of the state in which it does its largest volume of business. E) The bank is considered a citizen of Washington, D.C. and also of the state in which it does its largest volume of business.

B

Maria's strategy to try to change the law through petitioning city council, which others have done in the past, is rooted in which of the following? A) Legal positivism B) The historical school C) Legal realism D) Natural law E) Cost-benefit analysis

B

Mark and Jillian are business partners that decide to go their separate ways. As they are selling the business assets, Mark intentionally destroys two of the display cabinets that Jillian was taking with her to her new business. Jillian would sue Mark for ________. A) punitive damages B) compensatory damages C) nominal damages D) consequential damages E) property damages

B

Matthew, who attends a local state college in his home state, is prevented from wearing a T-shirt with a gun picture on it to his public school. He wants to sue because he believes his First Amendment rights are being violated. Which court could Matthew sue in? A) Federal court because diversity jurisdiction exists. B) Federal or state court because the case raises a federal question. C) The U.S. Supreme Court because the case is a First Amendment case. D) The state appeals court because the case raises a Constitutional issue. E) The federal appeals court because the case raises a Constitutional issue.

B

Miko comes to Jamal's law office to ask him about the law in regard to dogs running at large in her neighborhood. Jamal tells Miko that he will research the issue and get back to her. Where should Jamal look first in order to locate applicable law? A) Restatements B) State statutes C) Case law D) Uniform laws E) Executive orders

B

Montana implemented a new law that determined that males could not obtain their drivers licenses until they are 18. Females, on the other hand, could obtain their licenses at 16. If this law is challenged, what level of scrutiny would the courts use? A) Rational basis scrutiny B) Intermediate scrutiny C) Strict scrutiny D) Popular scrutiny E) Systemic scrutiny

B

Morgan sued Rachel over a motor vehicle accident, but they settled the case prior to the trial for $1,000. The lawsuit is now ________. A) Ripe B) Moot C) Cased D) Standard E) Remanded

B

Neville knows that he can bring his case against Ramona in a state court in Tennessee but he doesn't know the correct county in which to proceed. Which of the following address the proper county? A) In personam jurisdiction B) Venue C) Subject-matter jurisdiction D) Diversity jurisdiction E) Long-arm jurisdiction

B

Positive abstractions that capture our sense of what is good or desirable are called ________. A) Ethical ideas B) Values C) Conscience demands D) Desirable principles E) Action goals

B

Ronaldo promised to sell Linda a Dalmatian puppy for $700 but backed out of the deal. Linda sued Ronaldo in state court for breach of contract and asked for a jury to hear the case. During jury selection, one juror, Mika, said that she did not think she could be fair to Linda because Linda did not appear to be a dog lover. Linda's lawyer asked that Mika not hear the case on the basis that she could not be fair, and the judge excused Mika. Linda also decided that another juror, Sandy, looked at her in a grumpy manner so, without a reason given, Linda's lawyer asked the judge to excuse that juror from serving. The judge did so. After the jury was chosen, Linda's lawyer made a statement to the jury, as did Ronaldo's lawyer. Linda's lawyer then called to the witness stand Jules, a friend of Linda, who heard the discussion held between Linda and Ronaldo regarding the purchase of the puppy. Jules testified under questioning by Linda's lawyer that she heard Linda say that she would pay $700 for the puppy and that she also heard Ronaldo say that he would sell the dog for that amount. Unfortunately for Linda, Jules also testified in response to questioning by Ronaldo's lawyer that Ronaldo distinctly told Linda that he would only sell the puppy to her if Linda came with cash for the puppy within seven days. Linda did not show up with the money for ten days and Ronaldo had already sold the dog to someone else. The judge ruled in favor of Ronaldo. The challenge Linda's lawyer made to Mika is referred to as a[n] ________. A) Peremptory challenge B) Challenge for cause C) Stipulated challenge D) Fairness challenge E) Approved challenge

B

Sari, a wealthy Hollywood director, worked with the California legislature to create a law that banned actors and actresses from auditioning for roles outside of their gender. Under what test would a court analyze this law to determine if it violated the equal protection clause? A) Strict scrutiny, because of the gender-based classification. B) Intermediate scrutiny, because of the gender-based classification. C) Rational-basis, because of the gender-based classification. D) Strict scrutiny, because a fundamental right is involved. E) Rational-basis to determine if a fundamental right is involved.

B

Sofia, co-owner of a popular dance club called Salsa Sensations, discovered her co-owner and best friend, Renata, has been secretly pocketing cash receipts. Salsa Sensations' business is booming. Sofia meets Gordon, an attorney. Gordon advises Sofia that she should engage in ADR instead of filing a la Sofia tells Gordon that she wants to file a lawsuit against Renata, and then engage in mediation. Gordon tells her that she must mediate before filing a lawsuit. Is Gordon correct? A) Yes, once a lawsuit is initiated, she cannot engage in mediation. B) No, many courts have mediation programs. C) No, every court will mandate mediation after a lawsuit is filed. D) Yes, it is a waste of money to file a lawsuit before mediation. E) No, the Federal Arbitration Act mandates that mediation must occur.

B

Sofia, co-owner of a popular dance club called Salsa Sensations, discovered her co-owner and best friend, Renata, has been secretly pocketing cash receipts. Salsa Sensations' business is booming. Sofia meets Gordon, an attorney. Gordon advises Sofia that she should engage in ADR instead of filing a lawsuit against Renata because it would be faster and cheaper than litigation. 77) If Sofia wants to engage in ADR without the cost of paying a neutral third party, which type of ADR should Gordon suggest? A) Med-arb B) Negotiation C) Minitrial D) Early neutral evaluation E) Private trial

B

Which of the following is based on the idea that, when ruling on a case, judges consider more than just the law; they also weigh factors such as social and economic conditions? A) Cost-benefit analysis B) Legal realism C) Historical school D) Legal positivism E) Executive positivism

B

The Federal Aviation Administration has banned the presence of giant lizards on airplanes. Corina has a business training lizards for use in television and movies. Her star lizard, a massive iguana named Izzy, likes to have her own seat on the airplane. In order to promote her business, Corina has asked her home state of Montana to pass a law permitting giant lizards on airplanes. Corina believes the lizard ban by the Federal Aviation Administration (FAA) is not a law because it is an administrative agency rule. Is she correct? A) Yes, because the FAA is only an administrative agency. B) No, because federal laws include rules passed by federal administrative agencies. C) No, because of the supremacy clause. D) Yes, because there are no checks and balances on federal administrative agencies. E) Yes, because the FAA rules do not have the same status as laws.

B

The ________ contain(s) legislative acts passed by state legislatures. A) U.S. Code B) State codes C) Uniform Register D) State Reporter E) State Reference Manual

B

The ________ for ethical behavior seeks consideration of what the world would be like if a decision is copied by everyone else. A) Golden rule B) Universalization test C) Public disclosure D) Relevant disclosure E) World rule

B

The plaintiff must provide the defendant in a lawsuit with a copy of the complaint. That process is called ________. A) Summons issuance B) Service of process C) Service delivery D) Subpoena delivery E) In personam service

B

Trudy has moved from Texas to Alabama. The state does not allow nonresidents to seek employment without first passing a loyalty test to Alabama. Alabama has violated which clause? A) The right employment clause B) The privileges and immunities clause C) The non-resident's rights clause D) The commerce clause E) The dormant commerce clause

B

Under what theory, if any, could the president of Wild Music be held liable for Delilah's actions? A) Strict liability B) Responsible corporate officer doctrine, if he could have prevented the crime C) Absolute liability D) Mens rea E) None

B

What do courts generally presume regarding laws passed in accordance with states' police power? A) There is no presumption. B) The laws are valid. C) The laws are invalid. D) The laws are invalid unless it involves a First Amendment right. E) The laws are invalid unless it involves regulation of interstate commerce.

B

What does the Fifth Amendment's takings clause reference? A) The right of the government to take private property for public use without paying compensation to the owner. B) The requirement that the government pay the owner just compensation if the government wants to take private property for public use. C) The right of the government to put individuals in jail for up to ten days without a hearing. D) The right of the government to put individuals in jail for up to five days without a hearing. E) The requirement that the government compensate an owner for anything destroyed during a search.

B

What does the establishment clause of the First Amendment provide? A) Government cannot make a law prohibiting the free exercise of religion. B) Government cannot make a law establishing a religion. C) Government cannot make a law referencing religion in any manner. D) Government must establish laws respecting religion in order to prevent disruptive behavior such as terrorism and to allow free-exercise by the majority. E) That only religions recognized when the First Amendment was passed are entitled to recognition and protection.

B

What does the value of freedom mean? A) To possess a large enough supply of goods and services to meet basic demands B) To possess the capacity or resources to act as one wishes C) To treat all humans identically, regardless of race, class, gender, age and sexual preference D) To possess anything that someone is willing to grant you E) To maximize the amount of wealth in society

B

What is the North American Free Trade Agreement? A) A U.S. law which protects citizens of the United States from being required to follow rules of commerce in foreign countries. B) A trade agreement with Canada and Mexico to help establish the conditions of global trade. C) A trade agreement with 150 countries to establish the conditions of global trade. D) An agreement with the World Trade Organization which allows the WTO to adjudicate trade disputes. E) A U.S. law which mandates citizens of the United States must follow rules of commerce in foreign countries.

B

When a defendant makes a first appearance, which of the following occurs? A) A magistrate determines whether there was proof beyond a reasonable doubt for the arrest. B) A magistrate determines whether there was probable cause for the arrest. C) A magistrate determines whether the defendant was properly booked but not whether the defendant was properly arraigned. D) A magistrate determines whether the defendant was properly arraigned but not whether the defendant was properly booked. E) A magistrate determines whether the defendant was properly booked and also whether the defendant was properly arraigned.

B

When can an arbitration award be set aside? A) If the parties do not like the outcome of the arbitrator's award. B) If the award resulted from fraud, corruption or the arbitrator misbehaved in a way that prejudiced the rights of the parties. C) If the courts do not like the arbitrator that the parties picked. D) If the arbitrator was a lawyer and not a judge. E) If the arbitrator was a judge and not a lawyer.

B

When can government agents obtain a search warrant? A) When they can establish reasonable cause. B) When they can establish probable cause. C) When they can establish cause to a substantial certainty. D) Anytime they have an informant. E) Anytime the judge is in his office and they show identification as government agents.

B

Which claim would Jon have against Taylor for taking the lawn chair? A) Trespass to property. B) Conversion. C) Private nuisance. D) Negligence. E) Harassment.

B

Which of the following entities would be responsible to make new federal laws? A) State legislatures B) Congress C) Judges D) The federal courts E) The executive branch

B

Which of the following is an independent agency? A) The Federal Deposit Insurance Corporation B) The Consumer Product Safety Commission C) The General Services Administration D) The Small Business Administration E) The Office of Personnel Management

B

Which of the following is generally true regarding a storekeeper who detains a suspected shoplifter? A) There is no special rule for shopkeepers. Shopkeepers commit the tort of false imprisonment if they detain anyone even if that person is suspected of shoplifting. Instead, the police should be called. B) When a retailer has reason to believe that a person is guilty of shoplifting, the store may question the person about said shoplifting in a reasonable manner and for a reasonable length of time. C) Whenever a retailer has reason to believe that a person is guilty of shoplifting, the store may detain the person for any length of time but may not ask any questions prior to the arrival of the police. D) The shopkeeper may detain the shoplifter only for a reasonable length of time but may not ask any questions prior to the arrival of the police. E) The shopkeeper may detain a shoplifter but for no longer than 20 minutes because by law it is assumed that it should take no longer than that amount of time for law enforcement officers to arrive.

B

Which of the following is not a form of ADR? A) Mediation B) Litigation C) Negotiation D) Arbitration E) Summary jury trial

B

Which of the following is not an objective of tort law? A) To compensate innocent persons who are injured. B) To impose criminal penalties on those who commit wrongdoing. C) To contribute to an ordered society. D) To discourage retaliation by injured parties. E) To satisfy our collective sense of right or wrong.

B

Which of the following is true regarding privacy rights? A) The U.S. Supreme Court has said that the U.S. Constitution does not provide any right to privacy. B) The U.S. Supreme Court has found that U.S. citizens have a right to privacy based upon the U.S. Constitution. C) The U.S. Constitution specifically provides for a right to privacy within the Second Amendment. D) The U.S. Constitution specifically provides for a right to privacy within the Sixth Amendment. E) The U.S. Supreme Court has found that U.S. citizens have a right to privacy based upon the U.S. Constitution, and the U.S. Constitution specifically provides for a right to privacy within the Sixth Amendment.

B

Which of the following is true regarding the apprehension necessary for the tort of assault? A) Apprehension and fear are the same thing. B) A person may be in apprehension of physical harm, even if the person is too courageous to actually be afraid of that physical harm. C) Actual injury is necessary for valid apprehension to be proven. D) The test for assault involves subjective apprehension, not reasonable apprehension. E) There is no requirement that the apprehension be of immediate physical contact.

B

Which of the following occurs when an individual is confined or restrained against his or her will for an appreciable period of time? A) Forcible detention. B) False imprisonment. C) Qualified imprisonment. D) Contained detention. E) False detention.

B

Which of the following presents a problem when considering the theory of ethics known as absolutism? A) The questionable nature of the rules in most absolutist repositories seems overly flexible when applied to different situations. B) The unquestionable nature of the rules in most absolutist repositories seems overly inflexible when applied to different situations. C) The lack of objectivity. D) The fact that most individuals associated with this theory have been proven to have acted to the detriment of their followers in one way or another. E) The allegation that an excessive number of followers actually follow consequentialism.

B

Which of the following refers to the ability to understand the structure of what someone is saying and then apply a set of criteria to evaluate its worth? A) Absolute skills B) Critical-thinking skills C) Cost-benefit skills D) Reviewability skills E) Traditional lawyer skills

B

Which of the following refers to the basic fairness of laws that may deprive an individual of her life, liberty, or property? A) Procedural due process B) Substantive due process C) Independent due process D) The prohibition against taking E) The prohibition against pervasive regulation

B

Why does public figure privilege exist? A) To give special protections from defamation to public figures since they are in the public eye and their reputations matter more to them than the average person. B) To encourage free discussion about public figures. C) To discourage discussion about politicians so democracy can function in a more orderly fashion. D) To allow people to make up stories about politicians and celebrities because these public figures are people of interest to the public. E) To prevent people from making a profit by slandering public figures

B

Why have most courts struck down most hate speech codes and codes of conduct? A) The courts cannot agree that hate speech should be unconstitutional. B) The courts say most codes are too vague and broad. C) Lower courts are waiting for the U.S. Supreme Court to define hate speech. D) The courts are waiting for a concensus ruling from the United Nations on what hate speech is. E) The courts are waiting for Congress to implement hate speech legislation.

B

) Which amendment in the Bill of Rights guarantees a right to due process? A) First B) Second C) Fifth D) Sixth E) Seventh

C

A large company is about to lay off several people due to a struggling economy. The company must make some difficult decisions about laying off older workers who cost the company more money, or laying off the younger workers who cost the company less money but have the technical skills the company will ultimately need. The company is faced with a(n): A) Ethical misfortune B) Social responsibility C) Ethical dilemma D) Legal responsibility E) Unfortunate dilemma

C

A local Chamber of Commerce plans a seminar on "the social responsibility of business in our community." What does that term reference? A) The responsibility of business to make profit for shareholders. B) The responsibility of business to have annual meetings. C) The expectations that the community imposes on firms doing business inside its borders. D) The expectations of employees regarding wage rates. E) The expectations of management in regard to adequate utility resources.

C

A manager who deceives employees because he believes that the deception maximizes pleasure over pain is acting consistently with which ethical theory? A) Act deontology B) Ethical relativism C) Act utilitarianism D) Ethical fundamentalism E) Rule utilitarianism

C

A person who illegally accesses or enters another person's or a company's computer system to obtain information or steal money is which of the following? A) Backer B) Reacher C) Hacker D) Trespasser E) Illicit user

C

A treaty is a binding agreement between two states or between international organizations. If the executive branch negotiates a treaty it must then be approved by: A) The House of Representatives B) All 50 states C) The Senate D) The Supreme Court E) All of Congress

C

According to the text, which of the following would not be considered in the "WHO" in the WH process of decision making? A) Consumers and community B) Employees and management C) State and federal governments D) Communities and future generations E) Employees and owners

C

After a court action brought by Mary, Mary was awarded $1,000 in damages from Stanley because she proved by a preponderance of the evidence that Stanley damaged her bicycle. What type of law applied to Mary's case that resulted in her being awarded $1,000? A) Criminal B) Procedural C) Civil D) Natural E) Positive

C

Aiden, who just started at his new job as an assistant manager at a fast-food restaurant, wants to make a good impression. He thinks that things are going well. On only the second day on the job, however, he sees his manager, Debbie, slink out of the restaurant with a big box of hamburgers. She puts them in her car and speeds away. Aiden is fairly certain that Debbie did not pay for the hamburgers. Aiden asks advice of his best friend, Kyle; his girlfriend, Mia; his sister, Katie; and his dad, Joe. Kyle says that there are no real objective standards and that Aiden should just decide to do what is best for himself. Mia says that Aiden should think about the consequences and focus on the greatest benefit to all involved. She believes that it will do no real harm for Aiden to keep his mouth shut because the fast-food restaurant is making plenty of money and probably does not pay Debbie enough anyway. Plus, getting in trouble with Debbie could cause problems at work, and if Aiden loses his job, Aiden and Mia might have to put off their impending marriage. Katie says that on balance, following the rule of producing the greatest good would mean that he should report Debbie to her boss because stealing does not generally produce the greatest satisfaction. Aiden's dad says that as a matter of principle, Aiden should report Debbie to her boss because stealing is simply wrong. Which theory most accurately fits the advice given by Katie? A) Deontology B) Act utilitarianism C) Rule utilitarianism D) Ethical relativism E) Virtue ethics

C

An electrician working in a small town is the only electrician within a 100-mile radius. He knows that people who have medical equipment at home have the highest need for power, so he charges them double to fix problems with their electricity, compared with the average customer. The community is angry about this choice. This business practice is most likely: A) Ethical because owners should focus only on the impact of decisions on the firm. B) Ethical because it is not illegal for a company to set its prices based on maximizing profits. C) Unethical because businesses operate in a community and communities have expectations for behavior of individuals, groups, and businesses. D) Unethical because it is against the law for a company to charge one customer more than another. E) Unethical because there is only one morally proper thing to do in any situation.

C

An employer who cheats on his payroll and federal withholding tax because he believes that everyone has a right to cheat on their payroll tax would believe in a(n) ________. A) Utilitarianism B) Ethical relativism C) Categorical imperative D) Fundamentalist approach E) Deontology

C

Awards obtained under the United Nations Convention on the Recognition and Enforcement of Foreign Arbitral Awards: A) Are always upheld by all courts. B) Can never be enforced in US courts. C) Are recognized by most courts unless there is a defense to enforcing the award. D) Are first addressed at the United Nations for fairness. E) Are subject to international courts that review all awards.

C

Candy wants to start an Internet business which will operate domestically as well as internationally. She is told by the Chinese government that certain items on her site are objectionable and illegal, and that if she wants to do business in China, she must remove the objectionable material. Which of the following is true? A) Candy is within her rights and should stand her ground. She is a U.S. citizen, and so long as she obeys U.S. laws, she can do business in China. B) Candy is within her rights only if she petitioned her state senator and obtained his or her permission to proceed. C) If Candy wants to do business in China, she must abide by Chinese law. D) By international law, there is a set fee of $10,000 that Candy can pay if she wants to obey only U.S. law. If she pays that amount, she can continue in China without any modification. E) By international law, there is a set fee of $5,000 that Candy can pay if she wants to obey only U.S. law. If she pays that amount, she can continue in China without any modification.

C

Cars R Us is a manufacturer of car parts looking to expand their business outside of their current state with suppliers in other states. As part of their contracts with new parts dealers, their attorney would insert a ________ clause that will specify the law that will be applied to resolve any disputes under the contract. A) breach of contract B) venue C) choice-of-law D) change-of-law E) court choice

C

Citizens are protected from unreasonable searches and seizures under which amendment? A) First B) Third C) Fourth D) Sixth E) Tenth

C

Consumer Goods Corporation sells products that are poorly made. Tina, who has never bought a product from Consumer Goods, files a suit against the firm alleging that its products are defective. The firm could ask for dismissal of the suit on the basis: A) that the case is not ripe. B) that the case is moot. C) that Tina does not have standing. D) that Tina cannot sue a corporation. E) that only a state entity has jurisdiction to sue a corporation in its borders.

C

For purposes of diversity-of-citizenship, where does a corporation reside? A) The state of incorporation only. B) Only the state in which the corporation has its principal place of business. C) The state in which the corporation has its principal place of business and the state of incorporation. D) Any state in which the corporation does business. E) Any state in which the corporation has done business within the last five years.

C

Greg found out that Louise was using company funds to pay off gambling debts. Greg confronted Louise and was threatened by Louise that if he did not help him continue to skim company funds, he would have his friends "take care of Greg and his pretty little family" and showed him a gun. Once Louise is caught and charged with a crime, he also implicated Greg. Greg could claim which defense to his unwillingness to participate in the criminal activity? A) Fraud B) Necessity C) Duress D) Insanity E) Entrapment

C

How is absolutism different from ethical relativism and situational ethics? A) It applies utilitarianism. B) It holds that a cost-benefit analysis should be applied. C) It holds that whether an action is moral does not depend on the perspective of the person facing the ethical dilemma. D) It applies virtue ethics and concentrates on the accepted values of the person at issue as well as those of the community involved. E) It applies corporate ethics principles

C

If Isaiah claims he was acting on Lila's information which he believed to be public knowledge, which defense could he claim? A) Duress. B) Entrapment. C) Mistake-of-fact. D) The Fifth Amendment. E) Nolo contendere.

C

If a person is tried for murder and acquitted (found not guilty), which amendment protects against double jeopardy and the person not being tried again? A) The Third Amendment B) The Fourth Amendment C) The Fifth Amendment D) The Ninth Amendment E) The Tenth Amendment

C

If the city of Normandy passes a new municipal ordinance, what type of law would this be considered? A) Executive order B) Common law C) Statute D) Administrative law E) Local ordinance

C

In a trial, if the judge reports that there is a "hung" jury, the judge simply means that: A) The jury required the defendant to be retried. B) The jury wanted the punishment of the defendant to be a public hanging. C) The jury could not reach a verdict. D) The jury decided they did not have enough evidence and requests the trial to begin over so enough evidence can be produced. E) The jury was needing to talk to the judge in chambers to discuss the issues.

C

In order for a person to use the insanity defense in a criminal suit, a person must prove that their mental state was so impaired that they ________. A) could not distinguish right from wrong only B) could not understand the wrongful nature of the act only C) could not understand the wrongful nature of the act and could not distinguish right from wrong D) are on a medication that impairs their ability to understand their criminal actions E) the insanity defense is no longer allowed unless the crime is murder

C

In reference to the statements made to his friends, which of the following is true in regards to Ryan's remark that Melissa cannot recover because she has not suffered a loss of income? A) Ryan is correct. B) Ryan is correct only if he can show that he did not intend to cause Melissa loss of income. C) Ryan is incorrect because his statements constituted slander per se. D) Ryan is incorrect because special damages will be presumed. E) Ryan is incorrect but only because Melissa cannot establish libel per se.

C

In which of the following do parties select a neutral third party and explain their respective positions to this neutral, who then evaluates the strengths and weaknesses of the case? A) Summary jury trial B) Med-arb C) Early neutral case evaluation D) Private trials E) Neutral submission

C

What is the document called that informs the defendant of the lawsuit and explains that if the defendant does not respond to the lawsuit within a certain period of time, a default judgment will be entered? A) Complaint B) Answer C) Summons D) Instructional guide E) Transactional analysis

C

Judge Bob rules on a case in his state. A case in a neighboring state has the exact same facts. Judge Judy, when referring to Judge Bob's case A) Must follow the case as precedent B) Must follow the case as precedent but only if the law is exactly the same in her state C) May be influenced by the ruling but is not obligated to follow it D) Would have to seek guidance on whether or not she must follow it from a higher court in her jurisdiction E) Would need to seek guidance from a federal court in order to determine if she is obligated to follow Judge Bob's ruling

C

Jurisdiction refers to: A) A defendant's interest in having an impartial jury preside over a case. B) The plaintiff's interest in having a case heard in district court. C) The courts' power to render binding decisions on parties before them. D) The power of the plaintiff to file a lawsuit in any court of his choosing. E) The power of a defendant to have a case removed to an unbiased court.

C

Laws which enable the court to serve a defendant outside the state as long as the defendant has sufficient minimum contacts within the state and it seems fair to assert jurisdiction are called ________. A) Minimum contact statutes B) Significant contact statutes C) Long-arm statutes D) In rem statutes E) Quasi in rem statutes

C

Leona and Enoch lived next door to each other for over thirty years. In an effort to bring new business and jobs into the area, the Big City council planned to build a shopping center, business park, and hotel. Big City notified Leona and Enoch that it had the authority to seize their land for public use, and Big City would provide them temporary housing and $1,000 in moving expenses. Leona and Enoch filed suit. 84) Who would likely win this lawsuit? A) Leona and Enoch. They had owned the land for over thirty years. B) Leona and Enoch. Under the theory of eminent domain, the land must be used for "public purposes" and the land cannot be taken for economic development. C) Leona and Enoch. Even though the use of eminent domain for the purpose of economic development is constitutional, the government must pay the owner fair market value for the property. D) Big City. The use of eminent domain is constitutional. E) Big City. Economic development is a compelling government interest.

C

Lilly and Jorge are parties to a contract dispute. Lilly's attorney is questioning Jorge under oath and in the presence of a court reporter. This is known as a(n) ________. A) pleading B) interrogatory C) deposition D) inquisition E) production of information

C

Min-ji's threat to punch Daniel in the nose and her action in raising back her arm constitute a[n] ________. A) Assault and battery B) Battery C) Assault D) Justified retribution E) Negligence

C

Minimum contact requirements: A) Are the same in every state. B) Are used to define what personal service means. C) Were established in International Shoe Co. v. State of Washington. D) Prevent a defendant from ever being served while out-of-state. E) Do not include committing a tort within a state.

C

Of which of the following must a defendant be informed before questioning? A) That the defendant must answer questions, or there will be a presumption of guilt at trial. B) That statements made to law enforcement before arraignment may not be used against the defendant in court. C) That the defendant has the right to consult an attorney before speaking to the police. D) That unless the defendant can afford an attorney, the defendant will not have an attorney. E) That the right to an attorney does not begin until after the defendant is arraigned.

C

Sam was illegally growing marijuana in his home using heat lamps. The police randomly selected his street to search for illegal activity. Without any kind of a warrant, they used a thermal imager, an instrument that detects heat emissions, and determined that given the amount of emissions coming from Sam's house, he must be engaged in criminal marijuana cultivation. The police immediately broke down Sam's door, searched his home, and arrested him. The police officer in charge told Sam that he had no right to stay silent and that he might as well confess. Sam proceeded to confess to growing marijuana and a number of other crimes. The judge was so angry with Sam that he threw him in jail for two months without any kind of hearing. Sam's lawyer, who just graduated from law school, is not sure if the police acted legally in Sam's case by using the thermal imager. Sam's lawyer also questions whether Sam had a right to remain silent, and he is concerned that the judge has not granted Sam a hearing. Would Sam have a right to a hearing? A) Yes, due to Sam's right to substantive due process. B) Yes, due to Sam's First Amendment rights. C) Yes, due to Sam's right to procedural due process. D) No, Sam's Fourth Amendment rights were not violated E) No, due to Sam's Second Amendment rights.

C

Samantha creates photos featuring an unflattering photograph of the president of the college she attends and sells them outside of class for extra money. She did not obtain permission to use the photograph. Which of the following is true? A) Samantha has committed defamation of the college president in the form of slander. B) Samantha has committed defamation of the college president in the form of libel. C) Samantha has appropriated the photograph for commercial gain, which constitutes an invasion of privacy. D) Samantha has committed both defamation and invasion of privacy toward the college president. E) Samantha has not committed any tort.

C

Some laws have been adopted to help business conduct business online. Cyberlaw is based primarily on ________. A) International law B) Administrative law C) Existing law D) Natural E) Positive

C

Tamra, a first year law student, slipped on a freshly mopped floor while walking to class. She bumped into Anton, another first year law student, breaking his glasses. He was very angry with Tamra and slashed all of her car tires causing damages of $1,500. Anton also decided to sue Tamra for negligence claiming damages of $300 for his broken glasses. He decided that he already knew all about the law and did not need a lawyer. Anton sued Tamra in state court. Tamra, in the same lawsuit, brought an action against Anton for slashing her tires. At the trial in state court, Tamra told the judge that a friend, Aimee, told Tamra that she saw Anton slash all of Tamra's tires. The judge disallowed Tamra's testimony on that issue. Aimee also came to court and testified about seeing Anton slashing all of Tamra's tires. The state court judge ruled in favor of Tamra awarding her $1,500 in damages. Anton said that he was not giving up and that he would seek double damages on appeal in federal court. Tamra and Anton live in different states when not attending school. After the trial, Tamra reported Anton's action of slashing her tires to the police, who said that they would proceed with a criminal action against Anton. Anton goes to see Alex, a recent graduate who had just passed the bar, and asked Alex to represent him in a federal court appeal. Should the judge have disallowed Tamra's testimony about what Aimee told her about the tire? A) No, the judge was wrong and should have considered that testimony. B) Yes, the judge was correct to disallow the testimony because it involved a possible criminal action. C) Yes, the judge was correct to disallow the testimony because it was hearsay. D) Yes, the judge was correct to disallow the testimony but only because Aimee was in the courtroom and could testify herself; otherwise, it should have been allowed. E) Yes, the judge was correct to disallow the testimony but only because it purportedly came from an admitted friend of the defendant, not an independent witness; otherwise, it should have been allowed.

C

Taylor is angry because her neighbor, Jon, allows his dog to chase her cat, so she decides to get even. She moves a number of farm animals into her backyard and begins playing music at all hours of the night. She also steals a nice lawn chair from Jon's deck. She intends to keep the chair and not give it back. Taylor further takes a rake off of Jon's deck that she plans to return after she finishes raking her leaves. Jon did not give her permission to take the rake or the chair. Jon is not happy about the farm animals and the loud music. 83) Which of these would be the most appropriate cause of action, if any, against Taylor based upon the farm animals and the music? A) Trespass to property. B) Harassment. C) Private nuisance. D) Negligence. E) No cause of action is appropriate because Taylor is entitled to use her property as she sees fit.

C

The EU regulates taxes on Internet sales and the amount of pollution differently than the U.S. government. U.S. companies doing business in the EU: A) Are required to comply with EU regulations only if their main headquarters are located within the European Union. B) Must pay taxes to the EU when conducting all business globally if they wish to do any business in the EU. C) Must comply with EU regulations when doing business in the European Union only. D) Are never required to comply with EU regulations as long as they are headquartered in the United States. E) Have the option to conduct business under either EU or U.S. regulations

C

The Michigan legislature is contemplating a new tax law that would affect Julie's business. Julie consults an attorney in order to sue the legislature. What should the attorney tell her? A) That she does not have standing B) That the case is moot C) That the case is not ripe D) That she would have to sue in federal court E) That she does not have personal jurisdiction to bring the suit

C

The Sarbanes-Oxley Act was signed into law by: A) President Bill Clinton B) President Barack Obama C) President George Bush D) President Richard Nixon E) President Gerald Ford

C

The U.S. Courts of Appeals can hear appeals from ________. A) any federal district court B) neighboring district courts C) district courts in its geographic area D) the U.S. Supreme Court E) state supreme courts

C

The United States has strict standards for the production and transportation of chemicals. Dow Chemical insists that its suppliers conform to all the United States laws when they supply chemicals to other countries, even if the laws of these foreign countries are less restrictive in nature. This is a would be: A) Legal positivism B) Legal realism C) Natural law D) Cost benefit analysis E) Historical school

C

The ________ Amendment extends most of the provisions of the Bill of Rights to the states, prohibiting state interference in citizen's rights. A) First B) Tenth C) Fourteenth D) Twenty-first E) Twenty-fifth

C

The ________ of a firm are the many groups of people who are affected by the firm's decision? A) State governmental entities B) Federal governmental entities C) Stakeholders D) Corporate legal counsel E) Accountants

C

The ________ specifically provides the right to a speedy and public trial. A) First Amendment B) Fourth Amendment C) Sixth Amendment D) Due Process Clause E) arrest warrant

C

The city council is considering a new ordinance to require stores to provide reusable bags and eliminate disposal paper or plastic bags. In determining whether to pass the law, the city council obtained an estimate for the expense stores would incur in offering reusable instead of disposable bags. They also obtained an estimate for the savings which would result from not having to clean up litter from disposable bags. Finally, they consulted with an environmentalist to assess the positive impact that phasing out bags would have, including enhanced safety for local wildlife. In deciding whether to pass the law, the council relied on: A) The historical school B) Stare decisis C) A cost benefit analysis D) Common law E) Legal positive

C

The court hears a discrimination case in which they are asked to decide whether local anti-discrimination laws protect against a person being fired because they have red hair. The court holds that since Irish people are more likely to have red hair, this is an example of national origin discrimination and anti-discrimination laws should apply. The legislature, following the court ruling, passes a statute saying hair color is not covered by anti-discrimination laws. The legislature's action is an example of: A) Overturning stare decisis B) Overturning precedent C) Revoking common law by new statutory law D) Following precedent E) Following stare decisis

C

The ethical system in which the development of virtues, or positive character traits such as courage, justice, and truthfulness, is the basis for morality is called ________. A) Absolutism decision B) Virtual analysis C) Virtue ethics D) Fundamentalist approach E) Categorical imperative

C

The group that urges states to enact model laws to provide greater uniformity of law is called the ________. A) Model Law on Uniform State Laws Organization B) Federal Uniform Law Commission C) National Conference of Commissioners on Uniform State Laws D) Model and Uniform Law Consortium E) Uniform Statutory Enforcement Commission

C

The state court system has exclusive jurisdiction over ________. A) Admiralty cases only B) Bankruptcy cases only C) Adoption and divorce cases D) Admiralty, bankruptcy, and copyright cases E) The state court system has no exclusive jurisdiction

C

Theodore violates the city's building code by not installing fire alarms on every floor. This is considered a(n) ________ which is usually punishable by a jail sentence of less than six months or a small fine. A) felony B) misdemeanor C) petty offense D) tort offense E) unacceptable offense

C

What are the three categories of intentional torts? A) Torts against decency, torts against reputation, torts against persons. B) Torts against public policy, torts against property, torts against reputation. C) Torts against persons, torts against property, torts against economic interest. D) Torts against persons, torts against businesses, torts against the government. E) Assault, battery, defamation.

C

What does the value of justice mean? A) To possess a large enough supply of goods and services to meet basic demands B) To possess the capacity or resources to act as one wishes C) To treat all humans identically, regardless of race, class, gender, age and sexual preference D) To be safe from those wishing to interfere with your property rights E) To maximize the amount of wealth in society

C

What law gives immunity to providers of interactive computer services for liability they might otherwise incur because of material disseminated by them but created by others? A) The Internet Communications Act of 2000. B) The Interactive Computer Services Protection Act of 2004. C) The Communications Decency Act of 1996. D) The Internet Communications Protection Act of 1998. E) The Blog Protection Act of 2001.

C

What type of tort occurs when the defendant takes an action intending certain consequences will result or knowing certain consequences are likely to result? A) Criminal B) Liability C) Intentional D) Negligent E) Strict-liability

C

When courts rely on precedent, they are obeying ________. A) Common analysis B) Res judicata C) Stare decisis D) In rem process E) Federal law

C

Where are the three independent branches of the federal government stipulated in the Constitution? A) The Bill of Rights B) The first three amendments to the Constitution C) The first three articles to the Constitution. D) The Tenth Amendment. E) The Preamble to the Constitution.

C

Where can you find local common law? A) State reporters B) Regional reporters C) The local courthouse and clerk's office D) The state administrative code E) Municipal ordinances

C

Which of the following describes the role of arbitration and mediation in e-commerce cases? A) Increasingly, litigants are using arbitration, but not mediation, to resolve disputes in e-commerce cases. B) Increasingly, litigants are using mediation, but not arbitration, to resolve disputes in e-commerce cases. C) Increasingly, litigants are using arbitration and mediation to resolve disputes in e-commerce cases. D) Arbitration is allowed by federal law in e-commerce cases, but mediation is not allowed. E) Mediation is allowed by federal law in e-commerce cases, but arbitration is not allowed.

C

Which of the following is an example of the imposition of strict liability? A) A robber pulls a gun to rob a bank teller. B) A burglar breaks into a home to steal jewelry. C) A business sells cigarettes to a minor. D) A minor shoplifts in a department store. E) An adult shoplifts in a department store.

C

Which of the following is an example of the system of checks and balances? A) Congress can appoint a federal judge. B) The President refuses to approve Congress' appointment of a federal judge. C) Congress passes an amendment to overrule judicial rulings. D) Congress declares a law unconstitutional. E) The Supreme Court refuses to approve the president's appointment of a Cabinet member.

C

Which of the following is an exception to the rule that administrative searches usually require search warrants? A) The regulatory exception B) The reasonable-search exception C) The pervasive-regulation exception D) The compelling-need exception E) The controlling-principle exception

C

Which of the following is the secretive and wrongful taking and carrying away of the personal property of another with the intent to permanently deprive the rightful owner of its use or possession? A) Robbery B) White-collar burglary C) Larceny D) Arson E) Criminal fraud

C

Which of the following is true regarding strikes under the National Labor Relations Act? A) A union must contact the State Mediation Consortium to attempt to mediate its demands before beginning a strike to achieve higher wages or better working hours. B) A union must contact the National Mediation Service to attempt to mediate its demands before beginning a strike to achieve higher wages or better working hours. C) A union must contact the Federal Mediation and Conciliation Services to attempt to mediate its demands before beginning a strike to achieve higher wages or better working hours. D) A union must contact the Judicial Arbitration and Mediation Services to attempt to mediate its demands before beginning a strike to achieve higher wages or better working hours. E) A union has no obligation to inquire into mediation before beginning a strike for any reason.

C

Which of the following is true regarding trials in Japan? A) The Japanese trial system has juries that function similar to juries in America. B) The discovery process in the Japanese court system is much simpler than that in America. C) Trial involves a series of discrete meetings between the parties and the judge. D) Known evidence must be automatically exchanged without request or order of the court. E) Only civil penalties may be imposed in civil cases for parties violating judicial orders.

C

Which of the following occurs when someone engages in outrageous, intentional conduct likely to cause extreme emotional distress to the party toward whom the conduct is directed? A) Negligent infliction of emotional distress. B) Intentional infliction of strict-liability distress. C) Intentional infliction of emotional distress. D) Reckless invasion of solitude. E) Psychological infliction of distress.

C

Which of the following would fall under the state court's jurisdiction? A) Admiralty cases B) Patent and trademark claims C) Adoption D) Bankruptcy E) Federal criminal prosecutions

C

Wild Music's confidential client lists also contain financial and credit information for their clients. Delilah e-mails the client list to her home e-mail so that she can analyze and sell information about her clients' spending habits. Delilah's actions could constitute a violation of what law? A) RICO B) The M'Naghten Rule C) The Computer Fraud and Abuse Act D) The Foreign Corrupt Practices Act E) The Mail Fraud Act

C

Zach gets angry because Austin earned a better grade on a test than he did. They get into an argument, and Zach takes a swing at Austin, intending to hit him. Austin shoves Zach in order to avoid the blow. Which of the following is true regarding Austin's actions? A) Austin has committed a battery. B) Austin has committed both an assault and a battery. C) Austin has not committed an assault or a battery because he acted in self-defense. D) Austin cannot rely on self-defense because he was involved in the argument and not completely innocent. E) Austin cannot rely upon self-defense because his life was not in danger.

C

________ is a dispute resolution process in which the parties agree to mediation and if the mediation is unsuccessful on one or more points, the parties move on to arbitration. A) Mini jury trial B) Summary jury trial C) Med-arb D) Factual summary hearing E) Litigated dispute trial

C

________ is one type of consequentialism. A) Deontology B) Categorical imperatives C) Utilitarianism D) Relativism E) Activism

C

________ requires that we evaluate the morality of an action by putting ourselves in the position of the person that is facing the ethical dilemma. A) Positive realism B) Ethical relativism C) Situational ethics D) Kantian ethics E) Relativity ethics

C

After trial, the jury found the prosecution had met its burden of proof by a preponderance of the evidence. Did the jury apply the correct standard of proof? A) Yes, in a criminal trial, the prosecutor must persuade the jury by a preponderance of the evidence that the defendant committed the crime. B) Yes, because the jury is the proper finder of fact in a criminal trial. C) No, because the judge is the proper finder of fact in a criminal trial. D) No, in a criminal trial, the prosecutor must persuade the jury beyond reasonable doubt that the defendant committed the crime. E) No, in a criminal trial, the judge must find a preponderance of evidence against the defendant.

D

Aiden, who just started at his new job as an assistant manager at a fast-food restaurant, wants to make a good impression. He thinks that things are going well. On only the second day on the job, however, he sees his manager, Debbie, slink out of the restaurant with a big box of hamburgers. She puts them in her car and speeds away. Aiden is fairly certain that Debbie did not pay for the hamburgers. Aiden asks advice of his best friend, Kyle; his girlfriend, Mia; his sister, Katie; and his dad, Joe. Kyle says that there are no real objective standards and that Aiden should just decide to do what is best for himself. Mia says that Aiden should think about the consequences and focus on the greatest benefit to all involved. She believes that it will do no real harm for Aiden to keep his mouth shut because the fast-food restaurant is making plenty of money and probably does not pay Debbie enough anyway. Plus, getting in trouble with Debbie could cause problems at work, and if Aiden loses his job, Aiden and Mia might have to put off their impending marriage. Katie says that on balance, following the rule of producing the greatest good would mean that he should report Debbie to her boss because stealing does not generally produce the greatest satisfaction. Aiden's dad says that as a matter of principle, Aiden should report Debbie to her boss because stealing is simply wrong. 67) Which theory most accurately fits the advice given by Kyle? A) Deontology B) Act utilitarianism C) Rule utilitarianism D) Ethical relativism E) Virtue ethics

D

As far as the letter to the newspaper is concerned, which of the following is true in regards to Ryan's statement that Melissa cannot recover because she cannot show loss of income? A) Ryan is correct. B) Ryan is correct only if he can establish that he did not intend to cause economic loss to Melissa. C) Ryan is incorrect because his actions would be considered libel per se. D) Ryan is incorrect because general damages would be presumed. E) Ryan is incorrect because Melissa has proven special damages.

D

Bank teller Hugo receives $1,000 from a customer for deposit into the customer's bank account. Instead of placing the money into the customer's account, Hugo puts it into his pocket. Which of the following offenses has he committed? A) Forgery B) False entries C) False token D) Embezzlement E) Cramming

D

Bethenny, who resides in Rhode Island, breached a contract entered into in Florida with Geraldo. Geraldo successfully obtains a judgment against Bethenny for $150,000 in a Florida state court. Bethenny lives in a nice apartment in Rhode Island, and her only substantial asset is a vacation beach house in North Carolina. Which of the following is true regarding Geraldo's ability to obtain funds from a sale of the beach house? A) Geraldo is only entitled to obtain funds from property located in Florida, the location of the lawsuit, and cannot benefit from the property in North Carolina. B) Geraldo is only entitled to obtain funds from property located in Florida, the location of the lawsuit, or from Rhode Island, Bethenny's domicile, and cannot benefit from the property in North Carolina. C) Geraldo will be able to obtain his judgment from the sale of the beach house in North Carolina through in rem jurisdiction, and he is entitled to keep any funds obtained in excess of amounts owed. D) Geraldo will be able to obtain his judgment from the sale of the beach house in North Carolina through in rem jurisdiction, but he must return to Bethenny any funds obtained in excess of amounts owed. E) Under federal law, Geraldo will only be able to obtain his judgment from the sale of the beach house through in rem jurisdiction if after two years Bethenny has failed to satisfy the judgment, and any sums received in excess of amounts owed to Geraldo go the Federal Housing Administration.

D

Bob and Tom were in a car accident on a street within Normandy city boundaries. Bob was ticketed for driving under the influence (DUI). This was Bob's third DUI. Tom sustained bodily damages and damage to his vehicle. If Bob is sued by Tom and the city of Normandy: A) Bob will be sued under the criminal laws only of Normandy by both parties. B) Bob will be sued under the civil laws of Normandy only by both parties. C) Bob will be sued by Tom under the criminal laws of Normandy and the civil laws of Normandy by the city's prosecutor. D) Bob will be sued by Tom under the civil laws of Normandy and the criminal laws of Normandy by the city's prosecutor. E) Only Tom would be able to be sue Bob because of the accident.

D

Bob's action against Ricardo for defamation involves which types of law? A) Criminal law and public law B) Civil law and public law C) Insider law and public law D) Civil law and private law E) Administrative law and public law

D

Bruno lives in Maryland and obtained a valid judgment against Jorge. The ________ requires a court in Vermont to uphold a valid judgment obtained in Maryland by Bruno. A) The privileges and immunites clause B) The dormant commerce clause C) The preemption doctrine D) The full faith and credit clause E) The commerce clause

D

By using UPS to send correspondence and samples of the bands to patients, Dr. Eli committed which of the following offenses, if any? A) False pretenses B) Fraudulent use of transit C) Defalcation D) Mail fraud E) He is not guilty of any offense

D

Cases heard by the Supreme Court of China (Taiwan) are presented to how many judge(s)? A) 15 B) 9 C) 18 D) 5 E) 1

D

Certain types of crimes, such as violations of regulatory statutes, allow punishment without the proof of a guilty mind. This is known as ________. A) actus reus B) a negligent action C) mens rea D) liability without fault E) minimal harm

D

Cruel and unusual punishment is prohibited under the ________ Amendment. A) Second B) Fourth C) Fifth D) Eighth E) Tenth

D

Under the ________ test people are asked to consider what the world would be like were our decision copied by everyone else. A) Propositions values B) Profit motive C) Globalization D) Golden Rule E) Universalization

E

DeShaun, who is a bit eccentric, is fed up with the way a certain employer in his town treats employees and decides to sue on behalf of all those employees. DeShaun also says that he is going to start his case at the appellate court level, skipping over all those "lesser" judges. DeShaun says that those justices will surely hear him out and that he will also seek a jury. Regardless of the court involved, can DeShaun act as plaintiff for the employees? A) Yes, so long as he gets permission slips from all of them. B) Yes, so long as no employee files an objection. C) Yes, so long as he gives any money he receives to them. D) No, because he lacks standing. E) No, because venue is lacking.

D

Ethical decision making has which of the following values? A) Maximization of profits B) Justice and profit maximization C) Freedom, profit maximization, security D) Freedom, security, justice, and efficiency E) Freedom, security, and profit maximization

D

Geromy sends a letter to Melinda seeking money for his dying child. Geromy has no dying child and intends to use the money for personal pleasure. This is an example of ________. A) false entries B) false token C) false pretenses D) mail fraud E) defalcation

D

How was the process of judicial review established under which courts review legislative and executive actions to determine whether they are constitutional? A) By vote of the U.S. Senate B) By vote of the U.S. House of Representatives C) By vote of both the U.S. Senate and the U.S. House of Representatives D) By early common law E) By executive proclamation

D

If Maria decides to ignore the leash law and let dogs run freely in the fenced outside area, this is an example of which legal school of thought? A) Legal positivism B) The historical school C) Legal realism D) Natural law E) Cost-benefit analysis

D

If a United States company is determining whether or not to move their manufacturing plant to a foreign nation to help decrease costs but potentially could help the foreign nation build infrastructure and bring needed jobs to that region, the company would use which approach in their decision making? A) Natural law B) Common law C) Historical school D) Cost-benefit analysis E) Legal realism

D

If a company refuses to bribe a foreign official, even though bribery is necessary to do business in the foreign country, because the company believes bribery is always wrong regardless of consequences, the company is motivated by which theory of business ethics? A) Ethical relativism B) Absolutism C) Consequentialism D) Deontology E) Profit motive

D

If a defendant enters a plea in which they do not admit guilt but agrees not to contest the charges further, this is known as: A) A partial defense. B) An Information. C) A nuisance plea. D) Nolo contendere. E) Plea of avoidance.

D

If a defendant is a corporation: A) No service of process is required. B) Only the president of a corporation can be served. C) Only the agent of the corporation can be served. D) Generally either the president of the corporation or an agent can be served. E) The corporation does not have to receive service.

D

If a manufacturing plant continues to allow toxins above the legal limit to escape into the atmosphere in violation of state law, which law is implicated? A) Private B) Common law C) Criminal D) Public E) Black letter

D

If a party does not abide by an agreement reached in mediation, which of the following is true of the remedy? A) Because mediation is a voluntary process, there is no legal remedy. B) The remedy is to return to mandatory mediation. C) The remedy is to proceed to arbitration. D) The remedy is to sue for breach of contract. E) The remedy is to proceed to a minitrial.

D

In China, the practice of using guanxi is an integral part of doing business both to those firms already in the country and those working to get their firms into the country. Guanxi is built on reciprocity meaning: A) You pay for another person's business debts to help them get back on their feet. B) That certain parts of businesses must be registered first with the government. C) Paying a bribe to an official is acceptable as part of the company's first obligation to enter the country. D) If someone does a favor for you, you will be expected to return that favor in the future including access to partnerships, contracts and government officials. E) Not paying a bribe and convincing a local company to illegally sell materials to them, since demanding bribes is unethical behavior.

D

In United States v. Lopez, the U.S. Supreme Court addressed the constitutionality of the Gun-Free School Zone Act. What did the Court rule? A) The Court ruled that the law was a valid exercise of congressional authority under the full faith and credit clause. B) The Court ruled that the law was a valid exercise of congressional authority contained within the Bill of Rights. C) The Court ruled that Congress exceeded its power under the Bill of Rights when it passed the act. D) The Court ruled that Congress exceeded its commerce clause authority when it passed the act. E) The Court ruled that the law was a valid exercise of congressional authority under the commerce clause

D

In refusing to testify at trial, Miles relied upon which amendment? A) The Second Amendment. B) The Third Amendment. C) The Fourth Amendment. D) The Fifth Amendment. E) The Sixth Amendment.

D

In the U.S. system of torts, torts are commonly classified as: A) Intentional, negligent, or criminal, not strict liability. B) Intentional, criminal, or strict liability, not negligent. C) Intentional, negligent, strict liability, or criminal. D) Intentional, negligent, or strict liability. E) Civil or criminal.

D

In the landmark case of Marbury v. Madison, the U.S. Supreme Court determined that ________. A) The Constitution established the three branches of government B) The President had executive authority to make law C) The duties of the legislative branch were divided between the two houses of Congress D) The courts had the power to determine whether or not legislative and executive actions were constitutional E) The supremacy clause gave all the powers to the federal government to set laws for the states

D

In what situations would a court most often strike down an arbitration clause as unconscionable? A) Contracts between merchants B) Contracts between corporations C) Contracts between businesses and their vendors D) Consumer and employment contracts E) Courts never strike down arbitration clauses as unconscionable

D

Jacob calls Nick on the telephone and threatens to come over and break his nose. Which of the following is true? A) Jacob's conduct constitutes a battery. B) Jacob's conduct constitutes an assault. C) Jacob's conduct constitutes both a battery and an assault. D) Jacob's conduct does not constitute an assault because there is no risk of immediate bodily harm. E) Jacob's conduct does not constitute an assault because there was no physical contact.

D

Judge George decides a case before his court that has the same set of facts as a case he heard four months ago. Judge George should apply ________. A) Procedural laws B) A different state statute C) Restatement D) Precedent E) Legal realism

D

Judge Jones is reviewing a cyberbullying case in her courtroom. When Judge Jones rules, in her pronouncement of the case, she discusses the law along with the importance of social conditions behind her ruling. She also is parting from past precedents in order to follow societal values in regards to cyberbullying. Judge Jones is using which school of jurisprudence? A) The historical school of law B) Legal positivism C) Natural law D) Legal realism E) Stare decisis

D

Kelly is sure that Juan has stolen her basketball tickets from her office since he was talking about how badly he wanted to go to the championship game earlier in the week. Kelly confronts Juan and he denies that he has been anywhere near Kelly's office. Kelly pushes Juan into the closet and locks the door, leaving Juan for hours. Kelly has committed what tort? A) Justified imprisonment. B) Qualified imprisonment. C) Reasonable detention. D) False imprisonment. E) No tort was committed, Kelly had the right to detain Juan.

D

Leaf, an employee of the California Department of Forestry, insisted on signing inter-office memoranda with his name and the initials "FOJT", an abbreviation for his church "Followers of Jesus' Trees". Leaf's supervisor, Milo, told Leaf there was a rule prohibiting displays of religion at work and insisted that Leaf cease placing the initials of his church on government paperwork. When Leaf refused to follow the work rule, he was terminated. 70) Leaf wants to file a lawsuit against the Department challenging his termination. What would a court most likely rule? A) For the Department because the work rule did not substantially burden Leaf. B) For the Department because it was only a work rule, and not a law. C) For the Department because it did not violate the First Amendment. D) For Leaf, because the work rule violated his free exercise of religion. E) For Leaf, because the work rule violated the establishment clause.

D

Leaf, an employee of the California Department of Forestry, insisted on signing inter-office memoranda with his name and the initials "FOJT", an abbreviation for his church "Followers of Jesus' Trees". Leaf's supervisor, Milo, told Leaf there was a rule prohibiting displays of religion at work and insisted that Leaf cease placing the initials of his church on government paperwork. When Leaf refused to follow the work rule, he was terminated. After his termination, Leaf returned to his office to pack his belongings. He uploaded a message on the Department's website, which stated that Milo was a thief and has stolen thousands of dollars from the State of California. Milo, who had not stolen money from the State, later sued Leaf for defamation. How would a court likely rule? A) For Leaf because he has an absolute right to free speech under the First Amendment. B) For Leaf because the rational-basis test applies. C) For Leaf because he did not use any lewd, profane, or obscene words. D) For Milo because the First Amendment does not protect defamatory speech. E) For Milo, but only if Leaf also used fighting words.

D

Lena receives notice of a lawsuit that indicates the suit is in the federal district court. She has heard about the Alternative Dispute Resolution Act and has been told she is required by law to use a form of ADR before she goes to court. Is this true? A) Yes, Lena must agree to mediation which is required by law in all federal district courts. B) Yes, Lena must agree to arbitration which is required by law in all federal district courts. C) Yes, Lena could pick either mediation or arbitration because some form of ADR is required by law in federal district court. D) Under the act, Lena must consider the use of ADR, but courts are not uniform in their implementation of the act. E) No, Lena is not bound to follow ADR and can go straight to trial and cannot be forced into any ADR process by law.

D

Lori and Tim are negotiating a new business deal that requires an interpretation of a law made by their state twenty years ago. Tim knows that there are cases regarding this law that have been tried in a court within their city's boundaries. If Tim relies on these cases as the law, he is referring to ________. A) Statutory law B) Administrative law C) Executive orders D) Precedent E) Constitutional law

D

Matt intends to throw a baseball at Jamal and hit Jamal in the head with the ball because he missed an easy fly ball hit to him in the field. Jamal is standing directly next to Sally in the dugout. Matt throws the baseball but hits Sally in the head instead of Jamal. What type of tort did Matt commit against Sally? A) No tort was committed, Jamal was Matt's target not Sally. B) The tort of negligence because Matt only intended to harm Jamal. C) An accidental tort because Matt struck Sally by mistake. D) An intentional tort because even though Matt meant to hit Jamal, he still caused harm by hitting Sally. E) A strict liability tort occurred because throwing a baseball is inherently dangerous.

D

Most of the amendments to the Constitution are applicable to corporations. The courts have said that: A) All states look at corporations as people anyway. B) Congress specifically added the words "corporations" to many amendments after the Bill of Rights. C) Corporations are taxed like people so the laws must apply the same. D) The laws imply that corporations are treated as artificial persons for most cases. E) The Fourteenth Amendment incorporates corporations as people.

D

One purpose of the law is to: A) Allow individuals to impose rules on others B) Require businesses and individuals to rely on the dependability of others C) Enable businesses and individuals to rely on the good will of one another D) Facilitate a sense that change is possible after a rational consideration of options E) Preserve existing social order and prevent change

D

Which of the following would typically not be part of a mediation between Sofia and Renata? A) Both parties agree to the selection of the mediator. B) Both parties provide their view of the case to the mediator. C) The mediator assists in finding a resolution. D) The mediator makes a binding ruling. E) The mediator assists in drafting the contract.

D

________ laws are those laws that are implemented by a legislative body. A) Administrative B) Common C) Executive D) Statutory E) Natural

D

Paula worked for a department store that specialized in women's high-end lingerie. As she was working one day she noticed a miniature light protruding from behind a mirror in the dressing room. As she moved the mirror to get a better look at the light, she noticed a recording device. Thomas, her supervisor, quickly told her that the recording device was a security camera, however, the recording device actually belonged to Thomas. Has Thomas committed a tort? A) No, he was protecting the store's merchandise. B) No, since he is a supervisor, he has the right to record people in the dressing rooms. C) Yes, Thomas has committed fraud. D) Yes, Thomas has committed the tort of intrusion on an individual's affairs. E) Yes, Thomas has committed the tort of false light.

D

Pauline, the president of Business Corp, falsified documents to make it appear as if she had been granted stock options on certain dates, and she selected the dates after the fact, picking dates on which the stock price was low. Which of the following offenses, if any, did she commit? A) Insider trading B) Pretexting C) Defalcation D) Stock option backdating E) She did not commit any offense

D

Raul sued Juan over an employment dispute at Raul's company. Juan was accused of selling the company's secret recipe for their specialty desserts. Juan however won the lawsuit. Raul would like to appeal because he believes that the jury did not properly look at all of the exhibits he presented at the trial. Would a court hear this appeal? A) Yes, all cases where a jury does not look at all the evidence is appealable. B) Yes, but only if both parties agree to the appeal. C) Yes, but only if all the witnesses are available. D) No, to be eligible to appeal, the losing party must argue prejudicial error of law. E) No, Raul was the plaintiff and the plaintiff is not allowed to appeal.

D

Regarding Ryan's legal problems in Winland, which is a true statement? A) Ryan can rely upon the federal statutory laws of the U.S. as a defense but not the U.S. Constitution. B) Ryan is entitled to all U.S. Constitutional rights in Winland because he is a U.S. citizen, but he cannot rely on statutory law. C) Ryan can rely on U.S. laws as a defense only if he obtains an opinion from the office of the U.S. attorney that his operations in the U.S. have strictly complied with the law. D) Ryan must obey the laws of Winland when operating a business in Winland and cannot rely upon U.S. laws as a defense. E) Ryan can rely upon the federal statutory laws of the U.S. as a defense. Additionally, Ryan is entitled to all U.S. Constitutional rights in Winland because he is a U.S. citizen.

D

The Court of Appeals for the Federal Circuit hears appeals from all but which one of the following courts: A) United States Court of Federal Claims B) Court of International Trade C) Patent and Trademark Office D) United States District Courts E) United States Court of Appeals for Veterans Claims

D

The Fifth Amendment's protection against self-incrimination encompasses which of the following? A) A person only has to be a witness against himself or herself if a felony is involved. B) A person does not have to be a witness against himself or herself if a first criminal offense is involved. C) A person does not have to be a witness against himself or herself if prison time is involved as opposed to a mere fine. D) A person does not have to be a witness against himself or herself in a criminal case. E) A person has to be a witness against himself or herself

D

The United States entered into a trade agreement with ________ known as NAFTA. A) Canada B) Mexico C) Canada and Russia D) Mexico and Canada E) Russia and Mexico

D

The ________ is a body of law significant to business activities including sales, banking, and warranty. A) Federal Business Code B) Model Business Code C) Uniform Transactional Model D) Uniform Commercial Code E) Marketing Transaction Code

D

The difference between robbery and burglary is: A) burglary occurs on the street outside of a building or dwelling while robbery occurs inside of a building. B) robbery occurs on the street outside of a building or dwelling while burglary occurs inside of a building. C) states often identify the two as the same crime. D) robbery is the forceful and unlawful taking of personal property while burglary occurs when someone unlawfully enters a building with the intent to commit a felony. E) burglary is the secretive and wrongful taking of personal property while robbery is entering a building to commit a crime.

D

The due process clauses establishes what two types of due process? A) Substantive and independent B) Procedural and independent C) Commerce and independent D) Substantive and procedural E) Commerce and historical

D

The federal court system has exclusive jurisdiction over ________. A) Admiralty cases only B) Bankruptcy cases only C) Federal copyright cases only D) Admiralty, bankruptcy, and federal copyright cases E) The federal court system has no exclusive jurisdiction

D

The idea that we should interact with other people in a manner consistent with the manner in which we would like them to interact with us is called the ________. A) Equalization Rule B) Ethical Realization Rule C) Silver Rule D) Golden Rule E) Ten Commandments Rule

D

The word "tort" means which of the following? A) It is an English word meaning "accidental." B) It is a German word meaning "malfeasance." C) It is a Latin word meaning "misfeasance." D) It is a French word meaning "wrong." E) It is a civil law term meaning "liability."

D

There are two types of negotiations that parties can enter into, they are: A) problem-solving negotiations and neutral negotiations B) neutral negotiations and adversarial negotiations C) golden rule negotiations and problem-solving negotiations D) adversarial negotiations and problem-solving negotiations E) golden rule negotiations and adversarial negotiations

D

Trevor goes door-to-door selling magazine subscriptions. Trevor knows, however, that he is simply taking money for subscriptions and has absolutely no intention of ever arranging for the provision of magazines. Which of the following offenses, if any, has he committed? A) Defalcation B) False entry C) False token D) False pretenses E) Ponzi scheme

D

Under the Sarbanes-Oxley Act, it is now a felony to willfully fail to maintain proper records of audits and work papers for at least ________ years. A) 2 B) 3 C) 4 D) 5 E) 6

D

Under the Sarbanes-Oxley Act, which of the following is true regarding the destruction of documents in a federal bankruptcy investigation? A) The destruction of documents in a federal bankruptcy investigation is a misdemeanor with possible sentences of up to 364 days in jail. B) The destruction of documents in a federal bankruptcy investigation is a felony with possible sentences of up to 364 days in jail. C) The destruction of documents in a federal bankruptcy investigation is a felony with possible sentences of up to 5 years in jail. D) The destruction of documents in a federal bankruptcy investigation is a felony with possible sentences of up to 20 years in jail. E) The destruction of documents in a federal bankruptcy investigation is a felony with possible sentences of life imprisonment.

D

What is the arbitrator's decision called? A) Arbitrator's Decree B) Arbitration decision C) Settlement D) Award E) Subsequent Decree

D

What type of tort occurs when the defendant is careless and acts in a way that subjects other people to an unreasonable risk of harm? A) Criminal B) Liability C) Intentional D) Negligent E) Strict-liability

D

When Miles was found not guilty, and the agent immediately arrested Miles again, the agent's actions would be found unconstitutional under ________. A) the freedom of prosecution theory B) the prosecutorial misconduct theory C) the single trial defense D) double jeopardy E) the twice tried theory

D

When a state supreme court issues a binding ruling on a matter of law, who must follow the precedent set by the state supreme court? A) The United States Supreme Court B) The U.S. Congress C) State legislatures D) Lower courts within the same state E) All lower courts in the United States

D

When the Supreme Court made its ruling on the legality of homosexual marriage, they considered social and economic conditions surrounding marriage rather than relying on precedent alone. This was an example of: A) Cost-benefit analysis B) Legal positivism C) Historical school D) Legal realism E) Executive positivism

D

When you read a case, you should: A) Find the facts, look for the issue, critically think about the issue, and determine if your reasoning matches the reasoning of the judge. B) Identify the legal issues, apply the facts to the legal issues, and evaluate the reasoning of the judge in light of the facts of the case. C) Consider the truth of the facts of the case, identify legal questions raised, review the decision of the judge, and determine if the judge's decision was justified by the rule of law. D) Find the facts, look for the issue, identify the judge's reason and conclusion, locate the rules of law that govern the reasoning, and apply critical thinking to the judge's reasoning. E) Identify the facts, apply critical thinking skills to determine the truths of the facts, and identify how the judge interpreted the facts in the case.

D

Which of the following allows a defendant to be found not guilty by reason of insanity if he or she did not understand the nature of the act? A) The irresistible impulse act. B) The involuntary act. C) The substantial capacity test. D) The M'Naghten test. E) The necessity defense.

D

Which of the following is an investment swindle in which high profits are promised from fictitious sources and early investors are paid off with funds raised from later investors? A) False entries B) False token C) False pretenses D) Ponzi scheme E) Pretexting

D

Which of the following is false about a private trial? A) Occurs after a case has been filed in court. B) Is voluntary. C) The trial is conducted in private to ensure confidentiality. D) Because the referee's report is filed with the trial judge, the trial is not confidential and becomes public record. E) A referee writes a report stating the findings of fact and conclusions of law, and the report is filed with the trial judge

D

Which of the following is helping more and more litigants in resolving e-commerce disputes? A) United States Supreme Court B) The Patent and Trademark Office C) Arbitrators International D) National Arbitration Forum E) International Forum of ADR

D

Which of the following is not necessarily considered an alternative to litigation? A) Mediation B) Arbitration C) Private trial D) Negotiation E) Early neutral case evaluation

D

Which of the following is reported in the Code of Federal Regulations? A) Statutes B) Executive orders C) Common law D) Administrative law E) Congressional orders

D

Which of the following is true for both arbitrations and trials? A) The decision of a judge is legally binding, but the decision of an arbitrator is not. B) Cross-examination of witnesses occurs in trials, but not in arbitrations. C) Parties are required to have a lawyer in both arbitrations and trials. D) The parties may introduce witnesses and documentation, may cross-examine witnesses, and may offer closing statements. E) Closing statements are not permitted during arbitration.

D

Which of the following is true in regards to the Fourth Amendment's protections for individuals? A) There are no protections associated with the Fourth Amendment, only suggestions. B) The safeguards against freedom of speech of an accused. C) The safeguards and protections of an incarcerated individual to religious freedoms while being detained. D) The safeguard against unreasonable searches and seizures along with restrictions on warrants. E) That an incarcerated individual will have the opportunity to a trial with a jury of his peers.

D

Which of the following is true regarding the right of the federal government to obtain business books and records? A) The government has an absolute right to business books and records so long as a subpoena is issued by a grand jury. B) The government has no rights to such records without a search warrant, and a grand jury subpoena is insufficient regardless of whether a sole proprietor or a corporate shareholder is involved. C) Obtaining books and records from a sole shareholder of a corporation and also from a sole proprietor based only upon a subpoena issued by a grand jury violates the right against self-incrimination. D) Obtaining books and records from a sole proprietor based only on a subpoena from a grand jury violates the owner's rights against self-incrimination, but seizing records from a sole shareholder of a corporation in that manner does not violate rights against self-incrimination. E) Obtaining books and records from a sole shareholder of a corporation based only on a subpoena from a grand jury violates the shareholder's rights against self-incrimination, but seizing records from a sole proprietor in that manner does not violate rights against self-incrimination.

D

Adherents of ________ believe that when one individual—the caregiver—meets the needs of one other person—the cared-for party—the caregiver is helping to meet the needs of all the individuals who fall within the cared-for party's web of care. A) Deontology B) Utilitarianism C) Absolutism D) Virtue ethics E) Ethics of care

E

After trial, Lila is found guilty, however, she feels the proceedings against her were unfair. Which of the following, if true, could be a violation of Lila's Sixth Amendment rights? A) The courtroom was not private. B) Lila's attorney was expensive. C) She had to listen to witnesses against her. D) The court clerk read the charges against her. E) She knew one of the jurors, Zoe, from high school, and Zoe always disliked her.

E

Because of the major accounting and fraud scandals in the early 2000s against major business such as Enron, WorldCom, Global Crossing, and Arthur Andersen, which of the following federal legislative actions was passed? A) RICO. B) The False Claims Act. C) The Imminent Harm to People Act. D) The Fraud Recovery Act. E) The Sarbanes-Oxley Act.

E

) Ashley is unhappy with her neighbor, Josh, because Josh is pressing charges against her for repeatedly trespassing on his property. She tells him that unless he drops the charges, she will bring an action against him for nuisance because he plays his stereo late at night at times. While the stereo does not bother Ashley because she is up late anyway, she thinks that will encourage Josh to drop the trespassing charges. Josh, however, continues to press charges, and Ashley proceeds to sue Josh for nuisance. Which of the following is true? A) Ashley is completely within her rights and has done nothing wrong because Josh really is playing the stereo late at night. B) Josh can sue Ashley for malicious prosecution and win only if he wins the nuisance action. C) Josh can sue Ashley for abuse of process and win only if he wins the nuisance action. D) Josh can sue Ashley for malicious prosecution and win regardless of whether or not he wins the nuisance action. E) Josh can sue Ashley for abuse of process and win regardless of whether or not she wins the nuisance action.

E

) Which of the following is true regarding administrative agencies? A) There are state administrative agencies, but no federal administrative agencies. B) There are federal administrative agencies, but no state administrative agencies. C) There are state and federal administrative agencies, but no local administrative agencies. D) Administrative agencies exist only at the local county level. E) Administrative agencies exist at the federal, state, and local level.

E

A company has to dispose of bio-hazardous waste products. Dropping them in the landfill anonymously would be the cheapest solution, but the company does not do this because it is frightened that other companies might do the same and its workers would end up breathing in or drinking pollutants. In considering the possible complicity, the company's decision was shaped by the: A) Public disclosure test B) Profit motive C) Golden Rule D) WPH Method E) Universalization test

E

A majority of citizens in a democracy can agree to permit certain authorities to make and enforce rules describing what behavior is permitted and encouraged in their community. These rules are what we refer to as the ________. A) Electoral college B) Community standard C) Democratic validation D) Stare decisis E) Law

E

A national restaurant chain must pay damages for breach of contract to a cooking oil supplier. What type of law is involved? A) Public law only B) Private law only C) Civil law only D) Public, private, and civil law E) Private law and civil law

E

A reporter receives information from several highly credible sources that a famous actress is pregnant and that the father of the baby is not her husband. The story is later proved untrue. The actress sues for defamation. Is she likely to be successful? A) Yes, because the printed story would be likely to damage the reputation of the actress. B) Yes, because the printed story turned out to be untrue. C) Yes, because pregnancy is a private matter even for public figures. D) No, because the allegations would be unlikely to damage a person's reputation. E) No, because the reporter did not act with actual malice.E

E

According to the text, which of the following may be a part of the "how" in the WH process of decision making? A) Public disclosure, universalization, security, and the Golden Rule B) Values, public disclosure, and security C) Profit maximization, security, and public disclosure D) Whistle-blowing, the Golden Rule, values, and public disclosure E) Public disclosure, universalization, and the Golden Rule

E

Becky has been paid by a group of business people to hack into a competitor's computer in order to create a network shutdown during the huge Christmas buying season. Becky would be considered what? A) An embezzler B) A slammer C) A crammer D) A computer modeler E) A cyber terrorist

E

By her method of payment, Claire is guilty of which offense? A) Ponzi scheme B) Insider-trading scheming C) Pretexting D) Check defalcation E) Check kiting

E

Crimes that are less serious and are punishable by fines or imprisonment of less than one year are considered to be ________. A) petty offenses B) felonies C) juvenile petty offenses D) tort offenses E) misdemeanors

E

Dalynda threatens to tell Raul's boss that he is gambling on the company's time if he does not pay her $5,000. This is considered ________. A) bribery B) intentional deceit C) corrupt practice D) exploitation E) extortion

E

DeShaun, who is a bit eccentric, is fed up with the way a certain employer in his town treats employees and decides to sue on behalf of all those employees. DeShaun also says that he is going to start his case at the appellate court level, skipping over all those "lesser" judges. DeShaun says that those justices will surely hear him out and that he will also seek a jury. Will an appellate court hear DeShaun's case? A) Yes, so long as a state trial court judge approves and certifies the case for the appellate court without holding a trial first. B) Yes, but only if DeShaun can prove that he would have had to wait at least a year for a trial at the trial court level. C) Yes, but only if DeShaun files his case in state court as opposed to federal court. D) Yes, but DeShaun is not entitled to a jury. E) No, because DeShaun is required to first file the case in the appropriate trial court.

E

How do law and business ethics relate to each other? A) Ethics is unrelated to the law because ethics answers only moral questions. B) Ethics entirely determines what the laws will be, as ethics is focused on finding one right answer to every question. C) The law establishes what is ethical, as anything which is legal is automatically ethical. D) The law establishes ethical rules and boundaries, because laws take multiple stakeholders into consideration. E) Law and business ethics serve as an interactive system, informing and assessing each other.

E

If someone is overly fearful, is that person assaulted every time he experiences apprehension? A) Yes, because assault only requires apprehension regardless of whether it is reasonable. B) Yes, because assault is judged on the basis of the victim's level of apprehension only. C) Yes, because it is necessary to take a person's level of fear into account when interacting with that person. D) No, because assault requires unwanted physical contact. E) No, because assault requires reasonable apprehension.

E

In the U.S. Supreme Court case of Miranda v. Arizona, the court established what? A) The right of a defendant to a 24-hour break from interrogation B) The right of the state to question the witness continually for 24 hours C) The right of a person to remain silent when being questioned until they are brought to trial D) The right to be represented by an attorney paid for by the state. E) The right of a person to remain silent and the right to an attorney

E

Kimora, a licensed veterinarian, was recently elected to the U.S. Senate. Kimora is very concerned about the lack of care for homeless cats and dogs. She proposes legislation whereby citizens of all states are to be taxed sufficiently to provide for no-kill animal shelters, and that citizens in states with higher than average incomes are to be taxed at a higher rate than citizens of other states. She also proposes that federal funds for state highways be denied to any state that fails to provide assistance to low-income citizens in covering pet vaccinations. José, another new senator, tells Kimora that, apart from the mandated federal income tax, Congress lacks the authority to tax states because the U.S. Constitution expressly reserves that right to the states. Sven, another senator, tells Kimora that Congress has no authority to link highway funds or any other funds with social welfare objectives. Devonne, a clerk in the Senate, tells Kimora that Congress is absolutely prohibited from taxing residents of one state at a higher rate than citizens of another state. Is Devonne's statement about taxing residents of different states correct? A) Devonne is incorrect. The U.S. Congress has complete discretion on taxation. B) Devonne is partially incorrect because before the U.S. Congress taxes one state's residents at a higher rate than citizens of other states, a rational relationship between the difference and governmental need must be shown. C) Devonne is incorrect because before the U.S. Congress taxes one state's residents at a higher rate than citizens of other states, an important need must be shown. D) Devonne is incorrect because before the U.S. Congress taxes one state's residents at a higher rate than citizens of other states, a disparity of income of over 30% between one state and another must be shown. E) Devonne is correct.

E

Kimora, a licensed veterinarian, was recently elected to the U.S. Senate. Kimora is very concerned about the lack of care for homeless cats and dogs. She proposes legislation whereby citizens of all states are to be taxed sufficiently to provide for no-kill animal shelters, and that citizens in states with higher than average incomes are to be taxed at a higher rate than citizens of other states. She also proposes that federal funds for state highways be denied to any state that fails to provide assistance to low-income citizens in covering pet vaccinations. José, another new senator, tells Kimora that, apart from the mandated federal income tax, Congress lacks the authority to tax states because the U.S. Constitution expressly reserves that right to the states. Sven, another senator, tells Kimora that Congress has no authority to link highway funds or any other funds with social welfare objectives. Devonne, a clerk in the Senate, tells Kimora that Congress is absolutely prohibited from taxing residents of one state at a higher rate than citizens of another state. Is Sven's statement about linking funds with social welfare objectives correct? A) Sven is correct. B) Sven is correct only if the state is making efforts to put social programs into effect and is not acting unreasonably in refusing to provide other needed services. C) Sven is partially correct in saying that Congress has no authority to link highway funds with social services, but any other funds may be linked to social services by executive order. D) Sven is partially correct in saying that Congress has no authority to link highway funds with social services, but other funds may be linked to social services by an act of Congress. E) Sven is incorrect.

E

The statements made to the jury by the lawyers immediately after the jury was chosen are referred to as ________. A) Direct statements B) Closing selection statements C) Jury statements D) Influential statements E) Opening statements

E

Kyla is working at her office one day when her very upset roommate Katrina calls. Katrina tells Kyla that she knows that Kyla stole her money and is sending her boyfriend Jacob to get the money back. Kyla locks herself in her office because she is fearful of Jacob. What torts did Katrina commit? A) Assault only. B) Battery only. C) Assault and battery. D) Assault, battery and defamation. E) None of the above specific torts occurred.

E

Leaf, an employee of the California Department of Forestry, insisted on signing inter-office memoranda with his name and the initials "FOJT", an abbreviation for his church "Followers of Jesus' Trees". Leaf's supervisor, Milo, told Leaf there was a rule prohibiting displays of religion at work and insisted that Leaf cease placing the initials of his church on government paperwork. When Leaf refused to follow the work rule, he was terminated. ) Before his termination, Leaf tells Milo that he is drafting a California law that would provide funding to his church because the church serves to protect trees and the environment in general. Milo tells Leaf that such a law would violate the establishment clause of the U.S. Constitution. Under what circumstances would a state law be permissible under the establishment clause? A) If it has a secular purpose. B) If it has a secular purpose and does not foster an excessive government entanglement with religion. C) If the government can prove either (1) it has a secular legal purpose, (2) has a primary effect of neither advancing nor inhibiting religion, or (3) does not foster an excessive government entanglement with religion. D) If the government can prove it does not foster an excessive government entanglement with religion. E) If it has a secular legal purpose, has a primary effect of neither advancing nor inhibiting religion, and does not foster an excessive government entanglement with religion.

E

Lowell and Nadine have a dispute in federal district court. Lowell indicates that since their dispute does not involve a clause in their contract that mandates any form of ADR, he and Nadine can choose whether to participate in an ADR process or not. Is this true? A) No, litigants at the federal level are required to use arbitration programs. B) No, litigants at the federal level are required to use the court's mediation process. C) Yes, Nadine and Lowell can determine whether or not they want to participate in a form of ADR. D) Yes, the US Supreme Court ruled that the Alternative Dispute Resolution Act is unenforceable by law. E) All federal district courts can decide whether litigants must use ADR and Nadine and Lowell should check with the district court in which their case is pending.

E

Patty sued Raphael for hitting her car, alleging that Raphael was texting at the time when the accident happened. Patty did not present hard evidence of texting, such as phone records. The jury held Raphael was not liable for the accident because Patty had not proved Raphael was texting, and that in fact it was Patty who failed to yield to Raphael. Patty should: A) Appeal and try to convince the appellate court that Raphael actually was texting. B) Ask the judge to grant a default judgment because the jury was wrong. C) Ask the Supreme Court to overturn the decision because she made clear Raphael was texting. D) Appeal to federal court to overturn the decision, which was made in state court. E) Move on with her life, as she was unable to convince the jury to side with her on a question of fact.

E

Peter brings in a baseball bat to hit Mary because he is mad at her. When he arrives, Mary is sleeping. He lines up the baseball bat with her head and whispers that he is about to take a swing. She does not wake up, and before he hits her, he changes his mind and quietly leaves before Mary realizes he has arrived. Mary later finds out and sues for assault. Is she successful? A) Yes, because she was at risk of immediate, offensive bodily contact. B) Yes, because Peter's actions caused apprehension of immediate, offensive bodily contact. C) Yes, because Peter intended to place Mary at risk of unwanted and offensive contact. D) No, but she would have been successful in suing for battery. E) No, because she was sleeping and thus not in fear or apprehension of an immediate, offensive bodily contact.

E

Ronaldo promised to sell Linda a Dalmatian puppy for $700 but backed out of the deal. Linda sued Ronaldo in state court for breach of contract and asked for a jury to hear the case. During jury selection, one juror, Mika, said that she did not think she could be fair to Linda because Linda did not appear to be a dog lover. Linda's lawyer asked that Mika not hear the case on the basis that she could not be fair, and the judge excused Mika. Linda also decided that another juror, Sandy, looked at her in a grumpy manner so, without a reason given, Linda's lawyer asked the judge to excuse that juror from serving. The judge did so. After the jury was chosen, Linda's lawyer made a statement to the jury, as did Ronaldo's lawyer. Linda's lawyer then called to the witness stand Jules, a friend of Linda, who heard the discussion held between Linda and Ronaldo regarding the purchase of the puppy. Jules testified under questioning by Linda's lawyer that she heard Linda say that she would pay $700 for the puppy and that she also heard Ronaldo say that he would sell the dog for that amount. Unfortunately for Linda, Jules also testified in response to questioning by Ronaldo's lawyer that Ronaldo distinctly told Linda that he would only sell the puppy to her if Linda came with cash for the puppy within seven days. Linda did not show up with the money for ten days and Ronaldo had already sold the dog to someone else. The judge ruled in favor of Ronaldo. When Linda's lawyer questions Jules, it is called ________, and when Ronaldo's lawyer questions Jules, it is called ________. A) Absolute examination; counter-examination B) True examination; analysis examination C) True examination; trickster examination D) Direct examination; interrogating examination E) Direct examination; cross-examination

E

Tim decides that he is in desperate need of money and decides to rob a bank. He is caught and arrested. What type of law would be applied to him when brought to trial? A) Civil B) Procedural C) Tort D) Consumer E) Criminal

E

Tort law consists of which of the following? A) A civil or criminal action in state court. B) A civil or criminal action in federal court. C) A breach of contract. D) A wrong or injury to another, including a breach of contract. E) A wrong or injury to another, other than a breach of contract.

E

Under the Sarbanes-Oxley Act, which of the following is not true? A) It is a felony to willfully fail to maintain proper records of audits. B) Audit and paperwork associated with the audit must be kept for at least 5 years. C) Punishment can be up to 10 years imprisonment if audit paperwork is not kept for 5 years. D) The punishment for securities fraud has been increased to 25 years. E) If found in violation of securities fraud a person can be imprisoned for life.

E

Sam was illegally growing marijuana in his home using heat lamps. The police randomly selected his street to search for illegal activity. Without any kind of a warrant, they used a thermal imager, an instrument that detects heat emissions, and determined that given the amount of emissions coming from Sam's house, he must be engaged in criminal marijuana cultivation. The police immediately broke down Sam's door, searched his home, and arrested him. The police officer in charge told Sam that he had no right to stay silent and that he might as well confess. Sam proceeded to confess to growing marijuana and a number of other crimes. The judge was so angry with Sam that he threw him in jail for two months without any kind of hearing. Sam's lawyer, who just graduated from law school, is not sure if the police acted legally in Sam's case by using the thermal imager. Sam's lawyer also questions whether Sam had a right to remain silent, and he is concerned that the judge has not granted Sam a hearing. 50) Did the police act within their rights by using the thermal imager? A) The police acted within their rights so long as they can show that they asked a judge for a warrant before using the thermal imager, but the judge refused. B) The police acted within their rights so long as they can show that the street was truly chosen at random for a search. C) The police acted within their rights so long as they can show that Sam had been in jail previously on a drug offense charge. D) The police acted legally so long as they can show that Sam had been in jail previously for any offense. E) The police violated Sam's Fourth Amendment rights in using the thermal imager.

E

Sofia, co-owner of a popular dance club called Salsa Sensations, discovered her co-owner and best friend, Renata, has been secretly pocketing cash receipts. Salsa Sensations' business is booming. Sofia meets Gordon, an attorney. Gordon advises Sofia that she should engage in ADR instead of filing a lawsuit against Renata because it would be faster and cheaper than litigation. Gordon advises that they seek mediation, and, if mediation is unsuccessful, then they should request arbitration or file a lawsuit. However, Sofia is concerned that any statements she makes at mediation could be used against her. What should Gordon tell her? A) Statements made in mediation can be used in arbitration or trial. B) Statements made in mediation can be used in trial, but not in arbitration. C) Statements made in mediation can be used at trial, but only as a means of introducing evidence not produced by the parties to the lawsuit. D) Statements made in mediation can be used in arbitration, but not trial. E) Statements made in mediation cannot be used in arbitration or trial.

E

State intermediate courts of appeal: A) Exist in every state B) Are analogous to the U.S. Supreme Court C) Have very limited jurisdiction D) Are courts of last resort E) Are analogous to federal circuit courts of appeal

E

Susan has lost her appeal in her state supreme court on a matter concerning a question of pure state law. She wishes to appeal to the U.S. Supreme Court. Susan: A) Can appeal because her case has already been heard in the court of last resort in her state. B) Cannot appeal because cases arising in state court can never be heard in federal court. C) Cannot appeal because cases arising in state court can never be heard in the U.S. Supreme Court. D) Cannot appeal because her case has not yet been heard in a U.S. District Court. E) Cannot appeal because the U.S. Supreme Court will not hear cases considering questions of pure state law

E

Tamra, a first year law student, slipped on a freshly mopped floor while walking to class. She bumped into Anton, another first year law student, breaking his glasses. He was very angry with Tamra and slashed all of her car tires causing damages of $1,500. Anton also decided to sue Tamra for negligence claiming damages of $300 for his broken glasses. He decided that he already knew all about the law and did not need a lawyer. Anton sued Tamra in state court. Tamra, in the same lawsuit, brought an action against Anton for slashing her tires. At the trial in state court, Tamra told the judge that a friend, Aimee, told Tamra that she saw Anton slash all of Tamra's tires. The judge disallowed Tamra's testimony on that issue. Aimee also came to court and testified about seeing Anton slashing all of Tamra's tires. The state court judge ruled in favor of Tamra awarding her $1,500 in damages. Anton said that he was not giving up and that he would seek double damages on appeal in federal court. Tamra and Anton live in different states when not attending school. After the trial, Tamra reported Anton's action of slashing her tires to the police, who said that they would proceed with a criminal action against Anton. Anton goes to see Alex, a recent graduate who had just passed the bar, and asked Alex to represent him in a federal court appeal. Anton asked Alex what his chances are regarding an appeal filed in federal court. What should Alex tell Anton? A) He should tell Anton that a federal appeal looks promising but he would not be able to get double damages. B) He should tell Anton that the federal appeal looks promising but he would have to prove Tamra was negligent. C) He should tell Anton that the federal appeal looks good if the law school admits fault in leaving water on the floor. D) He should tell Anton that the federal appeal is not allowed because there are not enough damages at issue. E) He should tell Anton that a case from a state trial court would not be appealed to federal court.

E

The Economist published an interview of 500 top American business executives, who listed four factors they believe are essential for profits and corporate citizenship to function as partners. These four factors are: A) Compliance with community standards at all levels, focus on profit above ethics, community engagement, and a focus on fairness. B) Ethics at all levels, leadership at all levels, community engagement at all levels, and a focus on profit as the ultimate goal for all levels. C) Leadership at all levels, ethics at the upper level, community engagement at all levels, and positive employee/employer feedback. D) Ethics at all levels, leadership at all levels, community engagement at all levels, and private ownership. E) Leadership at all levels, employee engagement, rigorous measurement of achievement, and public-private partnerships.

E

The Federal Aviation Administration has banned the presence of giant lizards on airplanes. Corina has a business training lizards for use in television and movies. Her star lizard, a massive iguana named Izzy, likes to have her own seat on the airplane. In order to promote her business, Corina has asked her home state of Montana to pass a law permitting giant lizards on airplanes. Under what circumstances may Montana regulate the same subject matter as the federal government? A) Only when federal law expressly provides Montana with permission. B) Only if there is no federal law regarding the subject matter in the Montana state law. C) Only if the Montana law serves a compelling government interest. D) Never. A state cannot regulate the same subject matter as the federal government. E) When a person's compliance with Montana state regulation would not cause him or her to be in violation of a federal regulation.

E

The ________ stipulates that if a state tries to pass a law that conflicts within an area that has federal legislative jurisdiction it will be found unconstitutional. A) State powers doctrine B) Bill of Rights C) State preemption doctrine D) Privileges and immunities clause E) Federal preemption doctrine

E

The duties we owe others imply that humans have fundamental rights. This ________ is foundational to Western culture and asserts that whether a business decision is ethical depends on how the decision effects the rights of all involved. A) Cost-benefit analysis B) Rule of reason C) Situational realism D) Ethical relativism E) Principle of rights

E

Under which of the following circumstances would the government's search be considered "unreasonable" under the Fourth Amendment? A) Searching digital information on a cell phone with a warrant. B) Using a drug sniffing dog during a routine traffic stop with a warrant. C) Using a drug sniffing dog during a routine traffic stop without a warrant. D) Bringing a drug sniffing dog into someone's private home with a warrant. E) Bringing a drug sniffing dog onto someone's front porch without a warrant.

E

Unlike ethical relativism and situational ethics, ________ holds that whether an action is moral does not depend on the perspective of the person facing the ethical dilemma. A) Ethical positivism B) Moral responsibility C) Social responsibility D) Ethical absolutism E) Absolutism

E

What does the value of efficiency mean? A) To possess a large enough supply of goods and services to meet basic demands B) To possess the capacity or resources to act as one wishes C) To treat all humans identically, regardless of race, class, gender, age and sexual preference D) To be safe from those wishing to interfere with your property rights E) To maximize the amount of wealth in society

E

What type of torts occur when the defendant takes an action that is inherently dangerous and cannot ever be undertaken safely, no matter what precautions the defendant takes? A) Criminal B) Liability C) Intentional D) Negligent E) Strict-liability

E

What type or types of law are involved in the above situation, considering Hannah's interaction with both the officer and Dev? A) Public law but not private law B) Private law but not public law C) Civil law and private law but not public law D) Criminal law and public law but not private law E) Public law, private law, civil law, and criminal law

E

What was the result in the case of U.S. v. Park, the case in which the defendant, the president of a national food-chain corporation, claimed he should not be held liable for allowing food in a warehouse to be exposed to rodent contamination because he had delegated responsibility to subordinates? A) The court dismissed all claims against him because of insufficient proof that he had actively allowed contamination. B) The court dismissed all claims against him because of insufficient proof of negligence. C) The court refused to dismiss claims against him because the offense was a strict liability offense. D) The court refused to dismiss claims against him because of insufficient proof of delegation. E) The court refused to dismiss claims against him because of proof that he had responsibility and authority to prevent the contamination or to correct it and failed to do so.

E

When the organization in charge of proposing uniform laws proposes a statute, which of the following is true? A) All states must adopt the uniform statute within one year. B) All states must adopt the uniform statute within six months. C) The uniform statute automatically goes into effect without any action by state legislatures. D) A state legislature is not required to adopt the uniform statute; but if the state legislature decides to go forward with adoption, all portions of the statute must be adopted. E) A state legislature can ignore the proposed uniform law, adopt it in full, or adopt it in part.

E

Which of the following are defenses to an action for libel brought by a public figure in the United Kingdom regarding a statement made in the United Kingdom? A) Only that the statements were true. B) Only that the statements were made in Parliament. C) Only that the statements were made in court. D) That the statements were not made in Parliament or court. E) That the statements were true, that the statements were made in Parliament, or that the statements were made in court.

E

Which of the following are stakeholders of a business? A) Shareholders but not employees B) Employees but not shareholders C) Customers but not employees or shareholders D) Management but not employees E) Shareholders, employees, customers, and management

E

Which of the following could be a result of mediation? A) After the mediator issues a binding ruling, Sofia and Renata draft an agreement. B) After the mediator issues a binding ruling, Sofia can appeal. C) After the mediator issues a binding ruling, Sofia cannot appeal. D) The mediator could issue a binding ruling against Sofia. E) Sofia and Renata reach an agreement that could be enforced in court if either party breaches the mediation agreement.

E

Which of the following is a consideration in meeting the minimal standard for a firm to be considered an ethical business? A) Meeting the requirements of deontology. B) Meeting the requirement of the public disclosure test. C) Meeting the requirement of utilitarianism. D) Satisfying the company's board of directors. E) Considering the legality of a decision when choosing the means of production.

E

Which of the following is not a theory of business ethics. A) Ethical relativism B) Absolutism C) Consequentialism D) Deontology E) Profit motive

E

Which of the following is not a typical type of compensatory damage? A) Pain and suffering. B) Cost to repair damaged property. C) Medical expenses. D) Lost wages. E) Damages to punish the defendant for wrongdoing.

E

Which of the following is not an element of criminal fraud? A) False representation of a material fact B) The representation was meant to deceive C) There was reasonable reliance on the false representation D) There must be damages E) The damages must be $500 or greater

E

Which of the following is not an example of compensatory damages? A) Payment for the plaintiff's medical bills for surgery resulting from the injuries caused by an intentional tort. B) Payment for medical bills for pain killers the plaintiff was prescribed to cope with the pain from an intentional tort. C) Payment for the pain the plaintiff endured as a result of an intentional tort. D) Payment for lost wages due to time the plaintiff had to take off from work due to injuries sustained as a result of an intentional tort. E) Payment for attorney fees the plaintiff incurred filing a lawsuit to get compensation for injuries resulting from an intentional tort.

E

Which of the following is not considered an advantage of ADR over traditional litigation as set out in the text readings? A) ADR is usually less expensive than litigation. B) ADR is structured to be flexible for the parties. C) ADR is less adversarial in nature. D) Businesses may not want to set a precedent through a court decision. E) Companies can hide their legal issues from the public.

E

Which of the following is true about how ethical conceptions shape business law and business relationships in different countries? A) Ethics does not shape business law or business relationships because law and ethics are separate and not interrelated. B) Business law and business relationships should be the same in all countries because ethics provides one correct answer to all questions. C) Business law should be the same in all countries because protecting the rights of shareholders is always the ethically proper course of action. D) Different ethical understandings exist in different countries, but fairness is the highest ethical standard so countries must create uniform laws. E) Different ethical understandings prevail in different countries, so ethical conceptions shape business law and business relationships uniquely in each country.

E

Which of the following is true regarding federal jurisdiction? A) There is no exclusive federal jurisdiction in civil matters. B) If a case falls within federal jurisdiction, it cannot fall within state jurisdiction. C) Some cases fall within both federal jurisdiction and state jurisdiction, but there is no exclusive federal court jurisdiction. D) Some cases fall within both federal jurisdiction and state jurisdiction, but that only occurs in criminal matters. E) Some cases fall within both federal jurisdiction and state jurisdiction, but the federal court system has exclusive jurisdiction over some cases.

E

Which of the following is true regarding rights of the citizens of Belarus? A) The constitution restricts movement outside of the country. B) The defendant in a criminal case has no protection from providing evidence against himself or herself. C) Citizens have the right to profess any religion, but they must profess some religion. D) The constitution limits the workweek to 30 hours. E) Citizens accused of crimes are presumed innocent until proven guilty.

E

Which of the following is true regarding state courts of appeal? A) States only have an intermediate court of appeal if there is no state supreme court. B) In states that do not have an intermediate court of appeal, appeals go to the federal court of appeals. C) In states that do not have an intermediate court of appeal, there is no right of appeal to any court. D) All states in this country have intermediate courts of appeal. E) Not all states have intermediate courts of appeal; in those states, appeals go to the state court of last resort

E

Which of the following is true regarding the manner in which the exclusionary rule is derived from the U.S. Constitution? A) It is specifically set forth in the Fourth Amendment. B) It is specifically set forth in the Fifth Amendment. C) It is specifically set forth in the Sixth Amendment. D) It is specifically set forth in the Eighth Amendment. E) No section of the U.S. Constitution specifically states the provisions of the exclusionary rule.

E

Which of the following is true regarding tools of discovery? A) The only tool of discovery is interrogatories. B) Depositions are the only tool of discovery available to plaintiffs, but defendants may use both interrogatories and depositions. C) Summary motions, interrogatories, and depositions are tools of discovery available to all parties. D) Interrogatories and depositions are tools of discovery available to all parties, but summary motions and requests to produce documents are not tools of discovery. E) Interrogatories, depositions, and requests to produce documents are all tools of discovery available to all parties.

E

Which of the following is true under the WH process of ethical decision making? A) The interest of consumers should always be put before employees and management. B) The interest of owners and partners rank the highest because they put the capital into the business. C) Community is always the last stakeholder to be evaluated in any ethical decision. D) Though owners always have the highest level interest in any ethical business decision, managers and employees should be valued equally, then shareholders and the community. E) One stakeholder should not be valued above another under the WH framework for business ethics.

E

Which of the following serves to combat bribery of foreign officials? A) The Foreign Official Bribery Act B) The International Bribery Prohibition Act C) The Fair Business Act D) The International Fair Business Practices Act E) The Foreign Corrupt Practices Act

E

Which of these would be a stakeholder in the above company? A) The community only B) The shareholders only C) Future generations only D) The community and shareholders only E) The community, shareholders, and future generations

E

Which statement about state and federal court jurisdiction is true? A) State courts begin with exclusive jurisdiction until a federal court intervenes. B) In all cases, state courts have concurrent jurisdiction with the federal courts. C) Federal courts begin with exclusive jurisdiction until a state court intervenes. D) In all cases, state courts have exclusive jurisdiction unless the state's supreme court grants jurisdiction to a federal court in the state. E) In some cases, state courts have exclusive jurisdiction; in some cases, state courts have concurrent jurisdiction with the federal courts; and state courts also have the power to hear all cases not within the exclusive jurisdiction of the federal court system

E

Which statement is true regarding Chris's statement that the CEO could not be held liable for violations of the act? A) Chris is correct. Under no circumstances can a CEO be held personally responsible for violations under the act. Any fines would be imposed upon the business entity. B) Chris is incorrect. The act provides for harsh penalties, and a CEO who knows that the company's financial reports are incorrect but claims that they are truthful, can be heavily fined. There are no penalties, however, for destruction of financial documents. C) Chris is incorrect. The act provides for harsh penalties, and a CEO who destroys or changes financial documents to mislead can be heavily fined. There are no penalties, however, for misstatements of a company's financial reports because the company is solely responsible for its statements. D) Chris is incorrect, but any fine against a CEO under the act cannot exceed a nominal amount of $1,000. E) Chris is incorrect. The act provides for harsh penalties, and a CEO who knows that the company's financial reports are incorrect, but who claims that they are truthful, can be heavily fined. Additionally, a CEO who destroys or changes financial documents to mislead can be heavily fined.

E

Which statement is true regarding the act and Olivia's firing? A) The act does not provide protection for whistle-blowers such as Olivia. B) The act provides protection for whistle-blowers only if it can be shown that a significant amount of money, in excess of $5,000, was involved in the incident involved. C) The act provides protection for whistle-blowers only if it can be shown that a significant amount of money, in excess of $10,000, was involved in the incident involved. D) The act provides protection for whistle-blowers who work for an accounting firm, but not for any other employees. E) Olivia's whistle-blowing would be protected under the act, and her firing was illegal.

E

Yolanda found a better job and quit Robert's employment. Yolanda, however, wants to sue Robert because she believes that he may have terminated her employment based on age discrimination if she had not quit. At this point, which of the following would be Robert's best defense to a lawsuit filed by Yolanda in the state in which she worked? A) That Yolanda lacks standing. B) That the court lacks in personam jurisdiction. C) That the state's long-arm statute does not apply. D) That a default judgment is appropriate. E) That the case is moot.

E

You and your neighbor Hammy are arguing about trespass to realty. Which of the following situations is not associated with the tort of trespass to realty? A) When a person intentionally enters the land of another without permission. B) When a person intentionally causes an object to be placed on the land of another without the landowner's permission. C) When a person stays on the land of another when the owner tells him to leave. D) When a person refuses to remove something he placed on the property that the owner asked him to remove. E) When a person accidentally places something on the land of another.

E

Which of the following is true regarding the Uniform Mediation Act? A) There is no Uniform Mediation Act because of the controversy involved in attempting to negotiate such an act. B) It provides for a mediation privilege but does not specifically require that mediators identify conflicts of interest, and all states have adopted it. C) It does not provide for a mediation privilege but does specifically require that mediators identify conflicts of interest, and all states have adopted it. D) It provides for a mediation privilege but does not specifically require that mediators identify conflicts of interest, and less than half the states have adopted it. E) It provides for a mediation privilege, mediators are required to identify any conflicts of interest, and less than half the states have adopted it.

e


Kaugnay na mga set ng pag-aaral

Ecology Section II: Organismal Ecology; Water

View Set

CHAPTER 11 - Aggregate Demand I: IS-LM Model Construction

View Set

2.4 Integration and Strategic Management

View Set

Global Business Today 12e Assignment - Chapter 4: Differences in Culture

View Set